Sie sind auf Seite 1von 52

PEDIATRICS

CONTENTS
NEWBORN ................................................. .................................................. ............................ 7
GENETICS ................................................. .................................................. ................................. 83
DISORDERS OF CARBON HYDRATE METABOLISM ............................................... ....................... 93
NORMAL GROWTH AND DEVELOPMENT .............................................. ..................................................
149
NUTRITION AND MALNUTRITION ............................................... ................................................. 151
VITAMINS ..................................................... .................................................. .......................... 157
MINERALS AND ELEMENTS ............................................... .................................................. ... 177
RESPIRATORY SYSTEM ................................................ .................................................. ................. 185

IMMUNE FAILURES ................................................ .................................................. .......... 225


ROMATOLOGICAL DISEASES ................................................ .................................................. .. 235
INFECTIOUS DISEASES ................................................ .................................................. .... 245
VACCINES ................................................. .................................................. .................................. 291
PEDIATRIC CARDIOLOGY ................................................ .................................................. ......... 303
EMBRYOLOGICAL ABNORMALITIES ................................................... .................................................. ..
369
SMALL AND BIG INTESTINAL DISEASES ............................................. ........................................ 381
Liver, gallbladder and pancreatic diseases ........................................... ............... 401
HYPOTALAMUS AND HYPOPHYSIS DISEASES ................................................... ...................................
415
ERYTHROCYTE DISEASES ........................................................ .................................................. .........
463
LEUKEMIA AND LYMPHOMAS ............................................... .................................................. ........ 519
PAYMENT, PROTEINURY AND NEPHROTIC SYNDROME ............................................ ...............................
561
LIQUID-ELECTROLITE AND ACID-BASE BALANCE ..........................................
........................................... 595
CONGENITAL CENTRAL NERVOUS SYSTEM ABNORMALITIES, HYDROSEFALY, HEAD ENVIRONMENTAL
ABNORMALITIES. 601
NEWBORN
NEWBORN
PROBLEMS RELATED TO PERINATAL PERIOD
one. From the mother to the fetus in the etiology of which of the following neonatal diseases
do antibodies play a role? (September 2011)
A) Neonatal hypoglycemia
B) Neonatal hypocalcemia
C) Transient neonatal thyrotoxicosis
D) Macrosomy
E) Microcephaly
This question is very simple. The answer, of course, is
and transient neoantal thyrotoxicosis associated with the duration of antibodies. Neonatal graves' disease:
due to stimulant antibodies passed on to the baby due to the mother's disease
It happens.
• It is more serious if the antibody level is high. It usually occurs in active disease.
• If the mother is given antithyroid treatment, its appearance is delayed (starts after 3-4 days).
• If the thyroid receptor blocking antibody is also present, the symptoms may take several weeks.
• The findings improve with the decrease of antibodies passing through the mother in 6- 12 weeks. Rarely
persists.
Findings in neonatal gravity IUBG - PM Exophthalmos - wide open eyes Goiter (most common) Microcephaly
- Craniostosis - ventricular expansion Irritability, restlessness Triangular face Tachycardia, tachypnea,
Thrombocytopenia Hyperthermia HSM and jaundice Hyperactivity Severe hypertension
Findings to help diagnosis in prenatal period: It is thought with tachycardia and goiter in fetus.
Goiter is the most common finding with neonatal hyperthyroidism.

file:///C|/Users/User/Downloads/mcq.htm[04-Aug-19 10:20:40 AM]


(Answer C)
ALL TYPES OF TISTIME QUESTIONS
one
PEDIATRICS
2nd. Which of the following statements is true about what should be done to prevent bleeding due to
vitamin K deficiency in newborn babies? (December 2010)
A) There is no need for special prevention since vitamin K deficiency is not seen in breast-fed infants.
B) The mother is given a high dose of vitamin K before birth.
C) Multivitamin is started on the 15th day.
D) The baby is given 1 mg intramuscular vitamin K after birth.
E) Folic acid is started after the baby is born.
It was one of the easiest questions of the exam. Vitamin A and C levels in breast milk are good, while
vitamin K and D levels are low. These are also be-
are two vitamins given to burners. Vitamin K is given to everyone after birth with 1 mg IM K1. It should be
done for the prevention of classic hemorrhagic disease.
(Answer D)
3. Which of the following does not cause hypocalcemia in the newborn? (December 2010)
A) Maternal diabetes
B) Maternal anticonvulsant use
C) Prematurity
D) Hypomagnesemia
E) Maternal hypoparathyroidism
If you reverse it and solve it, a very easy question is, which makes hypercalcemia in the baby?
Maternal hypoparathyroidism is clearly seen. The use of maternal anticonvulsants caused vitamin K
deficiency by induction of enzyme
as well as vitamin D deficiency.
2nd
ALL TYPES OF TISTIME QUESTIONS
NEWBORN
Hypocalcemia in the newborn is divided into early and late according to the time of observation. Early
hypocalcemia of the newborn (first 4 days)
• DAB (PTH is low in uncontrolled diabetes, calcitonin and P are high, Mg is lost)
• It may be due to prematurity and low intake such as SGA.
• Perinatal stress (asphyxia)
• Alkalose (bicarbonate Ca bonds)
• Furosemide treatment (increases Ca excretion in urine and causes nephrocalcinosis) If citrate is used
during blood storage, Ca and Mg are bound by citrate.
Lipid infusion and phototherapy (white light) also causes hypocalcemia. Hypocalcemia in these conditions
occurs after the 5th day of the yacht. Late hypocalcemia of the newborn
• Excess phosphorus load (due to undiluted cow's milk, feeding is now very rare)
• Mg deficiency
• Vitamin D deficiency
• Pseudohypoparathyroidism
• Hypoparathyroidism
• Hypercalcemia in the mother
Hyperparathyroidism in the mother Hypercalcemia
The most common cause of the newborn is excessive calcium or vitamin D. Serum Ca level> 11 mg /
dl.
Hypotonia, lethargy, decreased absorption, hypotonia, constipation, polyuria (the effect of ADH on renal
tubules)
dehydration).
Reasons
• Iatrogenic excess Ca administration
• Excessive vitamin D administration
• Maternal hypoparathyroidism
• Subcutaneous fat necrosis
• Williams syndrome

file:///C|/Users/User/Downloads/mcq.htm[04-Aug-19 10:20:40 AM]


• Vitamin A intoxication
(Answer E)
4. A normal vaginal baby weighing 4000 grams is brought to the hospital because she cannot move her
right arm after birth. In the physical examination, it is found that the baby holds the right arm in extension
and internal rotation, there is little tonus on that side and there is no Moro reflection; however, there is no
pain in the baby during passive movements. Which of the following should be considered first in this baby?
(December 2010)
A) Absence or weakness of the isolated muscle on the affected side
B) Early signs of early onset muscular dystrophies
C) Humerus fracture on the affected side
D) Brachial plexus injury on the affected side
E) Since the baby is born 4000 grams, the diabetic mother may have a baby and thrombosis on the
affected side.
may have formed
ALL TYPES OF TISTIME QUESTIONS
3
PEDIATRICS
Macrosomia of the baby is significant for birth trauma. Although the absence of moro could occur in the
humeral fracture, it should have been painful. Brachial plexus is the most commonly used plexus at birth. in
this
most often C 5 - 6 is injured and the typical table is the Moro's failure and the waiter's hand. Erb –
Duchenne Paralysis: The most common is the brachial plexus paralysis. C5 - 6 roots are affected. Arm
name-
the forearm is in pronation (waiter's hand). There are no Moro, biceps and radial reflexes on the paralytic
side. Hand movements and hand-held are preserved.
(Answer D)
5. Which is one of the important factors that increase perinatal mortality
It is not? (September 2008)
A) Prematurity
B) Intrauterine growth restriction
C) Perinatal asphyxia
D) Placenta previa
E) Intrauterine infections
Nelson table question, causes of perinatal and neonatal mortality in fetus, preterm and term
given separately. Ablation placenta is counted among fetal causes. Placenta previa is not indicated as the
cause. While placental insufficiency is the most common cause of death in fetus, preterms are
they die from complications related. Asphyxia and birth trauma are seen as causes of death in term infants.
Congenital anomalies and infections are the cause of death in 3 cases.
Fetal deaths are more common in perinatal mortality than neonatal deaths.
Major causes of perinatal and neonatal mortality
FETAL PRETERM TERM
Placental insufficiency Immature Congenital anomalies
Intrauterine infections RDS Asphyxia / birth trauma
Severe congenital anomaly Intracranial hemorrhage Infections
Umbilical cord trauma Congenital anomaly Meconium aspiration
Hydrops fetalis Infections PPHT
Abruptio placenta NEC, BPD
(Answer D)
4
ALL TYPES OF TISTIME QUESTIONS
NEWBORN
6. In twin-to-twin transfusion syndrome developed in intrauterine period, donor fetus
Which of the following develops? (April 2008)
A) Polyhydramnios
B) Hydrops fetalis
C) Intrauterine growth retardation
D) Polycythemia

file:///C|/Users/User/Downloads/mcq.htm[04-Aug-19 10:20:40 AM]


E) Cardiac hypertrophy
The donor side is the artery side. The recipient is the vein side. The transmitter remains small while the
receiver is large. Considering that there may be those who have combined anemia and hydrops in their
minds, let us emphasize that hydrops are seen on the recipient side, which is polycythemic. Plesental
vascular anastomoses are more common in monochorionic twins. Feto-fetal transfusion syndrome develops
as a result of arteriovenous anastomosis. Hemoglobin 5 g / dl, 20% difference in body weight. Anastomosis
ablation is performed.
Arterial side-Transmitter Vein side-Receiver
Oligohydroamnios Polyhydroamnios
Small premature LGA premature
Malnutrition Hydrops
Pallor, anemia Pletorrhea, polycythemia
Hypovolemia Hypervolemia
Microcardia Cardiac hypertrophy, Myocardial dysfunction, Right ventricular outflow tract obstruction
tricuspid regurgitation, thick arteriole.
Hypoglycemia Large glomeruli
Thin-walled arterioles
Small or normal glomeruli
(Answer C)
7. Which of the following does not cause oligohydramniosis? (April 2006)
A) Fetal anomaly
B) Achondroplasia
C) Renal agenesis
D) Pulmonary hpplasia
E) Intestinal pseudoobstruction
Try to solve the question, which is the cause of polyhydramniosis. Amniotic fluid volume increases to <10
ml / day by the 34th week, then begins to decrease. Amniotic fluid is called <500 cc oligohydramniosis in
the third trimester and> 2000 cc polyhydramniosis. Membrane rupture should be excluded before the
diagnosis of oligohydramniosis. Oligohydroamnios may accompany congenital anomalies, IUGR (placental
insufficiency), severe renal anomalies, and drug intake that disrupt fetal urination. The most serious
complication is pulmonary hypoplasia. So pulmonary hypoplasia can both result and cause. Special attention
to drugs. New questions can be derived by replacing them with diuretics or other drugs.
ALL TYPES OF TISTIME QUESTIONS
5
PEDIATRICS
Polyhydroamnios preterm labor, abruptio placenta, fetal neuromuscular dysfunction and fetal swallowing
It is associated with GIS obstruction, which prevents the absorption of amniotic fluid.
Achondroplasia is the cause of polyhydramniosis.
Oligohydroamniosis Polyhydramniosis
Placental insufficiency Anencephaly-hydrocephalus
Preeclampsia GIS: Esophageal atresia, duodenal atresia, TEF, cleft palate and lip,
IUGG AC: Cystic adenomatoid malformation, diaphragmatic hernia, chylothorax
Fetal anomalies Achondroplasia, Spina bifida, Werding Hoffman, trisomy 18, 21, Klippel-Feil
Twin-twin transfusion (donor) Diabetic mother infant, Backwith-Wiedemann syndrome
Amniotic fluid leakage Fetal anemia, fetal heart failure, twin-twin transfusion (recipient), hydrops fetalis,
TORCH,
Renal agenesis / urethral atresia Polyuric renal disease
Prune-Belly syndrome Teratoma
Pulmonary hypoplasia
Amniotic nodosum
indomethacin
ACE inhibitors
Intestinal pseudo obstruction
(Answer B)
8. Which of the following conditions does Alpha Feto Protein (a-FP) not increase? (April 2001)
A) Trisomy 18
B) Anencephaly

file:///C|/Users/User/Downloads/mcq.htm[04-Aug-19 10:20:40 AM]


C) Spina bifida
D) Neural tube defect
E) Epidermolysis bullosa
It's the same kind of question. AFP is low. The opposite of such questions
the situation has to be in case. In trisomies, water is very low and protein is low. In other words,
polyhydramniosis and low AFP are seen.
Increased contact of the amniotic fluid with body surfaces or renal loss is prominent at the height of AFP,
Placental insufficiency and trisomies are prominent in low AFP.
Maternal serum alpha-fetoprotein level was determined in screening open neural tube defects at 15-18
weeks.
used. To remember that AFP, beta HCG and estriol were used in the triple test, trisomies had decreased
AFP.
it helps us understand more precisely.
(Answer A)
6
ALL TYPES OF TISTIME QUESTIONS
NEWBORN
High AFP
• Multiple pregnancy
• Open neural tube defects
Gastroschisis-omphalocele, intestinal atresia
• Congenital nephrosis,
• Epidermolysis bullosa
Low AFP
• Trisomy 18-21
• SGA
• Preclampsia
• Placental insufficiency
9. What kind of disease does the mother have when she has a heart block?
(April 97)
A) Diabetes B) SLE
C) Mitral disease D) Tuberculosis
E) Pyelonephritis
Heart block, neutropenia, anemia and thrombosis due to anti Ro and anti
topeni. Heart block is permanent, while others are temporary.
Let's remember that diabetes may be SGA and LGA, preeclampsia causes neutropenia and
thrombocytopenia.
Effects of Maternal Non-Infectious Diseases on Fetus and Baby
Cholestasis (HEV induced) Preterm labor
Cyanotic heart disease Intrauterine growth retardation (IUGR)
Diabetes mellitus Hypoglycemia, LGA, Severe Diabetes: SGA
Endemic goiter Hypothyroidism
Graves' disease / myasthenia gravis Neonatal transient thyrotoxicosis / transient myasthenia
Herpes gestationalis Bullous rash
Hyperparathyroidism Neonatal hypocalcemia
ITP / isoimmunthrombocytopenia or neutropenia Thrombocytopenia / neutropenia
Malignant melanoma Fetal tumor
Neonatal myotonic dystrophy, congenital contractures, respiratory Myotonic dystrophy
With genetic shortage, genetic antisypation, the symptoms become earlier and more severe.
Phenylketonuria CHD, Microcephaly, Mental retardation
Preeclampsia, eclampsia IUBG, Neutropenia, thrombocytopenia, fetal death
Renal Transplant IUBG
Blood incompatibilities Fetal anemia, hypoalbuminemia, hydrops, neonatal jaundice.
SLE Conj. Heart block, rash, anemia, thrombocytopenia, neutropenia
(Answer B)
ALL TYPES OF TISTIME QUESTIONS
7

file:///C|/Users/User/Downloads/mcq.htm[04-Aug-19 10:20:40 AM]


PEDIATRICS
10. Which is the cause of oligohydroamniosis? (April 94)
A) Esophageal atresia
B) Hydrocephalus
C) Neural tube defect
D) Renal agenesis
E) Gastroschisis
Renal agenesis is the cause of oligohydramniosis. See 7. Description of the problem.
(Answer D)
11th. Which drug causes jaundice in premature children? (April 94)
A) Vitamin K analogs
B) Cephalotin
C) Tetracycline
D) Aminoglycoside
E) Phenobarbital
Jaundice: Vitamin K, novobiocin causes jaundice in preterms. Sulfonamides de albumine
increases the risk of kernicterus.
Cephalotin Coombs positive hemolytic anemia Tetracycline: Enamel hypoplasia
Aminoglycosides: Deafness and nephrotoxicity.
(Answer A)
12. The most common cause of neonatal deaths in the first 24 hours in the newborn
Which is? (September 89)
A) Prematurity-immaturity
B) Birth trauma
C) Newborn asphyxia
D) Congenital malformations
E) Hypoxia
The highest risk of neonatal mortality occurs in infants born below 1000 g and before 28 weeks of
gestation. The lowest risk is between 3000-4000 gr and 38-42 weeks. Neonatal and infant mortality
The main determinant is low birth weight (occurs due to preterm birth or IUBGG). Low birth weight is
defined as below 2500 g. Prematurity is the most common cause in developed countries,
The most common cause in developing countries is intrauterine growth retardation.
(Answer A)
8
ALL TYPES OF TISTIME QUESTIONS
NEWBORN
EXAMINATION AND RESUSCITATION
one. Which of the following skin findings in the newborn spontaneously over time
It is not lost? (September 2011)
A) Port-wine hemangioma
B) Mongolian stains
C) Harlequin stains
D) Toxic erythema
E) Cutis marmorata
This question was similarly asked in previous TUS questions. Desired information is whether the
physiological and non-physiological skin lesions of the newborn are known. From this point of view, Port
wine hemangioma in A is definitely pathological and is typical for Sturge-Weber disease, a phakomatous
group disease.
Non-pathological skin findings of the beat:
• Capillary hemangioma: Most commonly located in the eyelid. Crying and fever become apparent. Those
who settle in the neck are called stork spots (Salmon spots).
• Vernix caseosa: Desquame is composed of epithelial cells, hairs and sebaceous secretion.
• Lanugo feathers: It is prominent in premature babies. The face is prominent on the back and shoulders.
• Acrocyanosis: Neonates may have peripheral cyanosis in the hands, soles of the feet and around the
mouth. It disappears within the first 48 hours.
• Mongol stain: It is a bluish-gray purple lesion with irregular edges, especially on the hips and sacrum. It
disappears in the first years of life.

file:///C|/Users/User/Downloads/mcq.htm[04-Aug-19 10:20:40 AM]


• Cutis marmaratus: It is a physiological introduction due to the change in ambient temperature due to lack
of neuromuscular maturation. It can be observed in the first few weeks. It can also be seen in seriously ill
infants.
• Harlequin discoloration (clown color): Half of the body is pink due to hyperemia and the other half is pale.
It is harmless and temporary. Harlequin baby OR is a type of severe ichthyosis.
• Milia: White yellow, pearl-like papules 1-2 mm in diameter. Sebaceous glands are caused by clogging. No
treatment is required.
• Miliaria: Rash. Occurs as a result of clogging of sweat glands due to excessive heat. Frequent bathing is
recommended.
• Toxic erythema: yellowish white in the middle, erythematous around, starting in 1-3 days, 7-10. days
disappear. It occurs on the face, trunk and extremities. Not on the soles of the hands and feet. Eosinophils
are detected by Wright stain.
• Pustular melanosis: Neutrophil-containing vesiculopustular rash on the cheek, neck, back, extremities,
hand and foot. There is at birth. They are benign lesions that disappear after 2-3 days leaving mild
hyperpigmentation. Both lesions should be distinguished from dangerous HSV and staphylococcal vesicular
diseases.
• Acne neonatarum: It occurs due to transplacental passing hormones.
ALL TYPES OF TISTIME QUESTIONS
9
PEDIATRICS
Pathological Skin Findings of Newborn:
• Edema: Localized edema: May be in Milroy and Turner syndrome. Generalized edema: Prematurity and
hypoproteinemia (erythroblastosis fetalis, nonimmune hydrops, congenital nephrosis or Hurler syndrome).
Mild edema is normally also seen.
• Cavernous hemangioma: Surface cystic hemangiomas. Kasabach Merrit syndrome (destruction of platelets
due to cavernous hemangioma and disseminated intravascular coagulation)
and Mafucci syndrome (multiple enchondromas and hemangioma).
• Port wine stain: Sturge Weber syndrome that fits the trigeminal nerve distribution and does not pass over
time.
muna accompanies.
• Scleremia: Hardening of subcutaneous adipose tissue due to sepsis and severe circulatory failure.
• Sacral dimple and hair: Spina bifida may be present.
• Bronze baby: It is seen as a result of phototherapy in direct hyperbilirubinemia.
• Petechiae, ecchymosis, purpura jaundice are other pathological skin findings. It is important that there is
no discoloration as a result of compression.
• Amniotic tape: Amputation, syndactyly, abdominal and chest wall defects may occur in the extremities as
a result of rupture of the antibiotic membrane and disruption of circulation in the tissues.
• Joint hypermobility and increased skin fragility: Ehlers Danlos, Marfan syndrome, congenital contractural
arachnodactilia, and other collagen defects.
(Answer A)
2nd. Which of the following does not cause microcephaly? (April 2010)
A) Fetal alcohol syndrome
B) Down syndrome
C) Congenital Rubella infection
D) Hyperthyroidism
E) Meningitis
Again, the question is similar to the old TUS question. Causes of microcephaly in the neurology department
of all TUS questions-
are listed. Here, the most disfigured meningitis is the cause of hydrocephalus, which can also cause
macrocephaly. The causes of hydrocephalus include meningitis. In this case, macrocephaly
We may think that the cause. Hyperthyroidism may also cause craniosynostosis. In this case, the head
circumference may be normal or small. But NELSON in the table of causes of microcephaly
meningitis and encephalitis, but not hyperthyroidism. Megalencephaly: Extremely large head circumference.
Hydrocephalus, cerebral gigantism (Sotos send-
rumor), storage diseases, metabolic diseases (such as glutaric aciduria, Canavan 's disease, Alexander' s
disease), neurocutaneous syndromes, familial. As you can see why the head circumference is large-
s hydrocephalus. On the basis of this, it can be thought that meningitis, which makes common

file:///C|/Users/User/Downloads/mcq.htm[04-Aug-19 10:20:40 AM]


hydrocephalus, will leave the head circumference large. However in the table clearly the causes of
microcephaly
Although it may not be canceled because it is a questionable question. Causes of hydrocephalus (including
subheading hydrocephalus)
• Holoprosencephaly
• Massive hydrocephalus
• Porencephaly
10
ALL TYPES OF TISTIME QUESTIONS
NEWBORN
Causes of microcephaly
Primary secondary causes
Familial (OR, OD) Radiation
Down syndrome TORCH infection
Edward syndrome Fetal alcohol / hydantoin
Cri-du-chat syndrome Meningitis / Encephalitis
Cornelli de Lange syndrome Malnutrition
Rubistein-Taybi syndrome Metabolic (Hyperphenylalaninemia)
Smith-Lemli-Opitz syndrome Hyperthermia
Hypoxic ischemic encephalopathy
Causes of hydrocephalus
Komunike Nonkomunike
Achondroplasia Aquaduct. Stenosis (X-linked, infectious)
Bacilli Impression Chiari / DandyWalker Malformation
Beningn subarachnoid space enlargement Klippel-Feil syndrome
Meningeal malignancy Mass lesion: abscess, hematoma, tumor, neurocutaneous
syndromes, Galena vein malformation
Meningitis / Posthaemorrhagic
Choroid plexus papilloma
(Answer D)
3. Which of the following can lead to a small front fontanel? (September 2009)
A) Craniosynostosis B) Mucopolysaccharidoses
C) Achondroplasia D) Osteogenesis imperfecta
E) Hypophosphatasia
Craniosynostosis is an early closure of sutures and fontanelles. Cases seen in small front fontanel:
1- Craniosinostosis, 2- Microcephaly 3- Congenital hyperthyroidism 4- Wormian bone anomaly Any ongoing
shape or size anomaly requires CT scan.
ALL TYPES OF TISTIME QUESTIONS
11th
PEDIATRICS
Wide front fontanel
Achondroplasia and Osteogenesis imperfecta Prematurity
Cleidocanial dysostosis Intrauterine growth retardation
Pycnodisostosis Hypothyroidism
Russel-Silver syndrome Hydrocephalus
13-18-21 trisomies Congenital rubella syndrome
Apert syndrome Rickets due to vitamin D deficiency
Hallermann-Streiff syndrome Hypophosphatasia
(Answer A)
4. Which of the following skin findings may be associated with other defects
considered pathological in the examination of the newborn? (April 2009)
A) Erythema toxicum B) Pustular melanosis
C) Lanugo D) Traces of amniotic tape
E) Mongolian stains
In the early period, the amniotic tape ruptures around the extremities like a thin gauze, causing
amputation, fibrotic band formation, and abdominal and chest wall defects. Amniotic membrane rupture
sequence is defined as dysruption in morphology. Toxic erythema is lesions involving eosinophilia. It is a

file:///C|/Users/User/Downloads/mcq.htm[04-Aug-19 10:20:40 AM]


lesion that develops after a few days and lasts for 1 week. Pustular melanosis is a neutrophil-containing
lesion at birth. HSV with Staphylococcal infections
Miscible. Lanugo are thin hairs that are abundant in preterm. Mongol stain is the most common lumbosacral
localization and gray-blue discoloration. A few years. If the edema is massive, it is pathological. Cutis
marmaratus persists because it is seen in peripheral circulatory disorders and some syndromes
becomes pathological. Scleremia is seen in infants with severe sepsis due to esterification of subcutaneous
adipose tissue.
is the finding showing the course.
(Answer D)
12
ALL TYPES OF TISTIME QUESTIONS
NEWBORN
Non-pathological findings:
• Toxic Erythema
• Temporary pustular melanosis
• Acne neonatorum
• Ebstein pearl, Ranula
• Mongolian stains
• Milia, Miliaria
• Capillary hemangiomas
• Harlequin color change
• Lanugo
• Verniks caseose
• Acrocyanosis Pathological findings:
• Scleremia
• Petechiae and ecchymoses
• au Cafe au lait ”stains, ction Junctional type”
• Jaundice, bronze baby
• Cavernous Hemangiomas
• Harlequin baby: OR, severe ichthyosis
• Port Wine stain
• Amniotic tape
• Sacral dimple
• Joint mobility changes
5. Kraniotabes is not an expected finding in which of the following situations?
(April 2007)
A) Prematurity
B) Marfan syndrome
C) Rickets
D) Osteogenesis imperfecta
E) Hydrocephalus
When craniotabes are pressed into the parieto-occipital region, the head collapses like a ping-pong ball.
Physiolo-
jima is. Premature lasts longer. Persistent craniotabes:
• Osteogenesis imperfecta
• Rickets, cretinism
• Cleidocranial disostosis
• Lacunar skull
• Down syndrome
• Marfan syndrome is a result of long stature, arachnodacty, lens sublux-
is an OD disease characterized by dissection and aortic dissection.
(Answer B)
ALL TYPES OF TISTIME QUESTIONS
13
PEDIATRICS
6. Which is not the cause of microcephaly? (April 2005)
A) CMV

file:///C|/Users/User/Downloads/mcq.htm[04-Aug-19 10:20:40 AM]


B) Radiation
C) Aquaductal stenosis
D) Trisomy 21
E) Fetal alcohol syndrome
Aquaduct stenosis causes macrocephaly by causing hydrocephalus. For detailed explanation
See.
(Answer C)
7. Which of the following is not one of the APGAR scoring criteria?April 2007)
A) Heart rate
B) Respiratory pattern
C) Muscle tone
D) Color
E) Patella reflex
Evaluation of APGAR
Parameter 0 1 2
Heart rate None <100> 100
No breathing Irregular-slow Regular, crying
Muscle tone Hypotonia Flexion of the extremities Active movement
NG catheter reflex answer No Grimace Cough sneeze
Color Blue - Pale Limbs Pink Blue, Body
Fully pink
Apperarance consists of Pulse, Grimace, Activity, Respiration.
APGAR is evaluated at the first and fifth minutes. The first minute Apgar score is not used to assess the
need for resuscitation. Heart rate, color and respiration in assessing the need for resuscitation
used. The fifth minute is used to evaluate the success of resuscitation. It does not predict the possibility of
cerebral palsy (CP). In most CPs, APGAR is normal. However, CP APGAR
develops more than normal ones in low. It is used to predict neonatal death in combination with umbilical
artery pH. 5. Minutes APGAR 0-3 is purple-
it is a better predictor of pH than pH 7.0. Apgar score 8-10: normal baby.
Apgar score 4-7: risky infant, tactile stimulation and oxygen may be given through a mask. Apgar score: <4
severe asphyxia.
(Answer E)
14
ALL TYPES OF TISTIME QUESTIONS
NEWBORN
8. Which of the following best describes Erb-Duchenne paralysis in infants?
(September 2004)
A) Arm weakness due to humeral fracture
B) Low due to wrist damage
C) Reduction in arm strength due to injury to selvical 5-6 nerves
D) Arm weakness due to clavicle fracture
E) Pseudoparalysis due to syphilitic arthritis
Erb-Duchenne Paralysis: The most common brachial plexus paralysis. Cervical 5th and 6th nerves are
damaged. The arm is in adduction and internal rotation and the forearm is in pronation (waiter's hand). On
the paralytic side Moro and
There is no biceps reflex. Hand movements and hand-held are preserved. The phrenic nerve (C3-5) may be
involved. Klumpke paralysis: C7-8, T1 roots are involved. Intrinsic muscles of the hand are involved. Claw is
the hand. There is no catch reflex. Moro is taken. Horner syndrome (ptosis and myosis) occurs when
sympathetic fibers of the T1 root are involved.
Phrenic nerve paralysis has difficulty in breathing. Breathing sounds are reduced on that side. Same on X-
ray
on the right side, and on USG, there is a decrease in breathing participation.
The facial nerve is the most frequently damaged cranial nerve.
(Answer C)
9. In the following cases, immediately after birth with mask and air sac
positive pressure ventilation is indicated? (April 2004)
A) Heart rate is 90 / min

file:///C|/Users/User/Downloads/mcq.htm[04-Aug-19 10:20:40 AM]


B) APGAR score of 8
C) Systolic blood pressure 60 mm-Hg
D) Hypotonia
E) Respiration rate of 20 / min
In cases where the heart rate is below 100, the mask and the air sac are irrespective of other findings.
and positive pressure ventilation.
Objectives of Neontal Resuscitation
To prevent morbidity and mortality caused by hypoxic ischemic tissue damage, adequate respiration and
heart
to beat.
Three parameters are used to determine the course of the resuscitation.
one. Respiratory effort; Respiratory rate and depth increase with tactile stimulation. Heart rate if inhaled
Evaluated.
2nd. Heart rate; is the most important determinant of resuscitation. Presence of adequate heart rate
does not rant. If the heart rate is below 100, positive pressure ventilation (PBV) is initiated.
3. Color; color may be discolored despite adequate ventilation and heart rate. If there is central cyanosis
oxygen should be given until it is determined. Oxygen is not required in peripheral cyanosis.
ALL TYPES OF TISTIME QUESTIONS
15
PEDIATRICS
PBV; inadequate respiration, increased cyanosis and / or CTA <100 / min despite oxygen administration
Used in the case.
Chest compression; KTA <60 / min or KTA despite 15-30 sec PBV application with 100% oxygen
It is done if there is no increase despite being between 60-80. Chest compression always
100% oxygenated PBV should be accompanied.
Endotracheal Intubation
one. If ventilation with mask and air sac is unsuccessful
2nd. If you need long-term PBV
3. If tracheal aspiration is needed
4. If there is suspicion of diaphragmatic hernia
makes
(Answer A)
10. Which of the following is not used in the prophylaxis of neonatal conjunctivitis?
(April 2002)
A) 1% silver nitrate
B) 1% erythromycin
C) 1% tetracycline
D) 2% povidone-iodine
E) 1% sulfomide
Silver nitrate is the most effective prophylaxis against gonorrhea. Povidone can be used in iodine.
Tetracycline and
Erythromycin is one of the drugs involved in prophylaxis.
When gonorrhea conjunctivitis develops, third generation cephalosporins are used in systemic treatment.
Erythromycin and tetracycline can be used for prophylaxis against chlamydia. However, if conjunctivitis
develops,
erythromycin should be given systemically in the treatment of deion. The purpose of this is to prevent
pneumonia.
l is to. Tetracyclines under the age of 8 are not used for treatment because they disrupt the development of
teeth and bones.
(Answer E)
16
ALL TYPES OF TISTIME QUESTIONS
NEWBORN
Conjunctivitis developing in the first day is usually shimic conjunctivitis due to AgNO3 use.
The conjunctivitis that started after day 2 was the most common causative agent N. gonorrhea. Most
commonly used treatments
silver nitrate drops or erythromycin pomades.

file:///C|/Users/User/Downloads/mcq.htm[04-Aug-19 10:20:40 AM]


Pseudomonas conjunctivitis 5-18. days.
11th. Which of the following is a pathological finding in the newborn? (September 2001)
A) Epstein pearls B) Sclerema
C) Milia D) Mongol stain
E) Cutis marmaratus
See description of question 3
(Answer B)
12. Heart rate 93, slow and irregular breathing rate, slightly flexed extremities,
nasal catheter insertion into the nose and body pink, purple limb
What is the APGAR score in the newborn? (April 2000)
A) 3 B) 4 C) 5 D) 6 E) 7
Each parameter is broken by 1 point. In APGAR, the maximum score can be 10 and minimum 0.
See the description of question 2.
I would like to remind you of information that has not been asked before. False positive and false negative
APGAR is a high-potential information.
False positive APGAR: Low in the absence of hypoxia or acidosis.
False negative APGAR: Normal detection despite acidosis and hypoxia.
Reasons for false positivity: Immature, narcotic-sedatives to mother, administration of magnesium sulfate,
acute
cerebral or spinal trauma, congenital myopathy, neuropathy, CNS anomaly, diaphragmatic hernia,
coanal atresia, sepsis, hemorrhage-hypovolemia and asphyxia during recovery
Reasons for false negativity: High fetal catechol amine level, maternal acidosis and some term infants
(Answer C)
13. Resuscitation immediately after birth in the baby, tactile stimulation spon-
It was found that respiration occurred and heart rate was 78 / min. Before this baby ...
Which of the following should be done? (September 1999)
A) Intratracheal adrenaline
B) IV NaHCO3
C) Positive pressure ventilation with mask and air sac
D) Breast pressure
E) Free O2
ALL TYPES OF TISTIME QUESTIONS
17
PEDIATRICS
Resuscitation is needed in 5-10% of all babies. The aim is to prevent hypoxic ischemic damage.
Resuscitation needs assessment: Respiration, heart rate and color are used. APGAR is not used.
A, B, C of resuscitation: A: Airway, position, aspiration, intubation. Breathing: Tactile warning, PBV
(mask pouch or tulle), C: Circulation: Breast compression and medication to maintain circulation.
D: Medicines
Resuscitation steps
Procedures to be performed in the first 30 seconds after birth:
one. The radiant heater is placed underneath.
2nd. Respiratory tract is opened (head is slightly extended, aspiration first mouth, then nose).
3. Drying (prevents heat loss, tactile warning).
4. The baby's condition is assessed: Respiratory-heartbeat and color.
Breathing is good, KTA> 100 and cyanosis is given O2, pink is not necessary intervention.
Positive pressure ventilation (PBV): Apnea, CTA <100, persistent cyanosis. PBV respiration
and KTA should not be terminated without recovery. If the heart rate is 60 - 100 / min, PBV is continued.
Chest compression: It is performed if KTA is <60 / min despite thirty sec PBV. Cardiac massage lower 1/3
of the sternum
section. Compression / ventilation should be 3/1. The speed should be 120 / min.
The drug is used if there is KTA <60 or asystole despite thirty-second effective PBV and compression.
To initiate breathing, tactile stimulation is used to flick the soles of the feet, stroking the back. Lack
tactile cautions: back hit, anal sphincter dilatation, thoracic compression,
hot / cold application, pushing legs towards belly, cold oxygen.
Drugs used in neonatal resuscitation:
one. Adrenaline: 1: 10000 at a concentration of 0.1-0.3 ml / kg iv or is given from the endotracheal tube.

file:///C|/Users/User/Downloads/mcq.htm[04-Aug-19 10:20:40 AM]


2nd. Volume expanders: Serum physiological, lactated ringer and blood.
3. Sodium bicarbonate: Used in proven acidosis or prolonged resuscitation. One-on-one
diluted 8.4% NaHCO3. Without adequate ventilation and oxygenation
should not be used. It may cause IVC and paradoxical cerebral acidosis.
4. Naloxane: Opioid antagonist. IM, IV or intratracheal.
Dopamine / dobutamine are other drugs that can be used in dextrose.
Unused: Atropine and calcium
(Answer C)
14. Which one does not have a large fontanelle? (September 1999)
A) Osteogenesis imperfecta B) Congenital hyperthyroidism
C) Hydrocephalus D) Rickets
E) Achondroplasia
Hyperthyroidism is the cause of small anterior fontanelle. See the description of question 3.
(Answer B)
18
ALL TYPES OF TISTIME QUESTIONS
NEWBORN
15. Vacuum delivered newborn sutures and discoloration
If swelling was detected which of the following should be considered? (September 96)
A) Caput succadenum B) Cephal hematoma
C) Subdural bleeding D) Subarachnoid hemorrhage E) Linear fracture
Hood succadeneum: Edema with scalp crossing the sutures and midline. Edema within 2-3 days
It disappears. Molding and ecchymotic color changes may be accompanied by sutures. Cephal hamatoma:
Subperiostal hemorrhage. It is limited to one bone and does not cross the other side of the suture. There is
no discoloration. It gives fluctuations. It often occurs a few hours after birth and becomes more
pronounced. It may be with a linear fracture. Depending on the size of the cephal hematoma resolves in 2
to 12 weeks. Subgaleal hemorrhages: Hemorrhage under the galea aponorotica. Cross the sutures. This
area can be massive due to the large potential gap. It may spread to the neck. Linear fractures consist of
suture diatase and rupture of emissary veins. It expands after birth and causes fluctuating mass.
Hypotension and consumption should be monitored for coagulopathy. Skull fractures: Caused by
compression of forceps or maternal symphysis pubis, sacral promontorium, or ischial spins. Linear fracture
is most common. It is asymptomatic and does not require treatment. If collapse fractures are excessive,
they should be corrected. Fractures in the occipital bone or separations in the basal and squamous part
always cause bleeding because it will tear. Intracranial hemorrhages: It is due to birth trauma, asphyxia
and coagulation disorder in term infants. Subarachnoid is subdural and subtentorial. Subarachnoid and
subdural hemorrhages are not seen by USG. CT or MRI is used. Xanthochromic CSF is detected in
subarachnoid hemorrhage. Subdural hemorrhage is an indicator of abuse except for the newborn. The ICC
in the preterm is periventricular-intraventricular and subepandimal germinal matrix.
They arise. USG is used for diagnosis.
(Answer A)
16. Which of the following is the causative agent of conjunctivitis in a 12-day-old baby? (September 96)
A) Chlamydia trochomatis B) N. gonorrhea
C) Chemical conjunctivitis D) P. auriginosa E) H. influenza
Ophthalmia neonatorum: It is the most common eye disease in the newborn. Redness, chemosis, edema of
the eyelids and discharge are seen. The emergence times give an idea about the factor. Silver nitrate
(chemical conjunctivitis): 6 to 12 hours after birth. There is clear discharge.
It is related to irritation and passes in 24-48 hours. N. gonorrhoea Incubation is 2-5 days, redness and
purulent discharge occurs within 24 hours. Treatment is delayed
however, it progresses to corneal perforation. C. trachomatis: Incubation is 5- 15 days. Tarsal involvement
occurs. There is sticky discharge. Corneal involvement is rare. The organism is the most common cause of
conjunctivitis. Pneumonia and eosinophilia suggest chlamydial infection. Pseudomonas: Makes a rare but
serious picture. Pannus formation can cause endophthalmitis, sepsis and death.
knows. N. gonorrhoea and pseudomonas can cause corneal perforation, blindness and death. Prophylaxis
uses silver nitrate (useless in active infection) Erythromycin and povidone iodine.
(Answer A)
ALL TYPES OF TISTIME QUESTIONS
19

file:///C|/Users/User/Downloads/mcq.htm[04-Aug-19 10:20:40 AM]


PEDIATRICS
17. What is your diagnosis if there is no other clinical pathology in the child using forceps at birth, internal
rotation of the left arm and extension of the left hand pro- cedure? (September 95)
A) Clavicle fracture B) Shoulder dislocation
C) Klumpke D) Erb Duchenne
E) Sepsis
See description of question 7
(Answer D)
18. Conjunctival hyperemia in the first 6 hours after birth in a newborn child
If a clear discharge is detected in the eye, which of the following should be considered? (September 95)
A) Clamidial conjunctivitis B) Gonococcal conjunctivitis
C) Chemical conjunctivitis D) Physiological conjunctivitis
E) HSV Type II conjunctivitis
See explanation of question 13
(Answer C)
19. The presence of systemic disease in a newborn infant
indicates the presence of? (September 94)
A) Sclerema B) Ebstein Pearls
C) Milia D) Mongol stain
E) Transient malignant pustular lesion
Scleremia occurs in severely ill children and children with sepsis.
(Answer A)
20. Which is not one of the APGAR score parameters? (September 94)
A) Reflex activity B) Blood pressure
C) Heart rate D) Skin color
E) Muscle tone
See description of question 6
(Answer B)
21. Which of the following does not cause hearing loss in the newborn? (April 94)
A) Perinatal asphyxia B) Caesarean section
C) Hearing loss in the family D) Birth below 1500 grams
E) Use of forceps
Indirect hyperbilirubinemia is a risk for deafness. Diseases that cause direct hyperbilirubinemia are not a
risk. Remembering of ototoxic drugs, aminoglycosides, vancomycin and furosemide
They are needed. The question can be updated again by replacing it with another antibiotic. Jaundice is the
most common of meningitis due to streptococci.
20
ALL TYPES OF TISTIME QUESTIONS
NEWBORN
Newborns at high risk for deafness
Family history Asphyxia
Congenital infections Prolonged mechanical ventilation
Craniofacial anomalies Werdenburg syndrome,
Alport syndrome, birth below <1500gr
Hyperbiluribinemia, Ototoxic drugs Bacterial meningitis Jervell - Lange - Nielson syndrome
Pendred and Usher syndrome
(Answer B)
22. Heart rate less than 90, respiratory superficial, slight flexion of extremities, nose
What is the APGAR score of the child who grimaced when the catheter was inserted? (September 92)
A) 1 B) 3 C) 5 D) 7 E) 9
See description of question 6
(Answer C)
23. Which of the following is pathological in a term newborn? (September 92)
A) Moro B) Babinski C) Capture D) Hypotonosity E) Stepping
The term newborn color is pink, in the semiflexion position.
Tonus: Defines resistance to passive movements. Preterm hypotonicity while term infant hypotonicity
not as a normal finding. Primitive reflexes are normally taken in the newborn.

file:///C|/Users/User/Downloads/mcq.htm[04-Aug-19 10:20:40 AM]


Their disappearance is craniocaudal. Suction and swallow coordination is 32-34 GH. Those born before this
week are fed with catheters.
Asymmetric Tonic Nuchal Reflex: In the supine position, when the child suddenly turns his head to one side,
it is the extension of the extremities and flexion of the opposite side. Criminal position.
It develops fully in 3-4 weeks after birth.
Standing capture is the latest disappearing reflectance. Parachute reflex: 7-8 months after birth begins to
be taken. Latest and never disappeared
It is a reflex.
Reflex Start (week) End times (months)
Moro 28-32 3-6
Suction 32-34 4-7
Search 32-34 3
Capture by hand 28 2
Standing capture 28 10
Tonic neck 35 7
(Answer D)
ALL TYPES OF TISTIME QUESTIONS
21
PEDIATRICS
24. Which of the following is true for Erb-Duchenne paralysis? (April 92)
A) C8-T1 B) C6, 7-T1
C) C7-8 D) C5-6 + Moro reflex loss
E) Loss of muscle strength in the lower extremity
Erb- Duchenne paralysis is defined as the waiter's hand. Brachial plexus is the most common injury. Moro
and
The biceps reflex disappears. Capture is preserved. See question 8
(Answer D)
25. A fluctuating swelling in the left parietal was detected in a 2-month-old baby.
What is the diagnosis? (September 91)
A) Caput quatratum B) Epidural hematoma
C) Subdural hematoma D) Cephalic hematoma
E) Caput sucsadeneum
Cephalous hematoma is associated with fracture. Subperiostal hemorrhage that does not cross sutures and
resorbs in 2-12 weeks
It happens. See explanation of question 12
(Answer D)
26. If a swelling was detected in the newborn that does not exceed the localized sutures
what? (April 90)
A) Cephalic hematoma B) Caput succadenum
C) Subgaleal bleeding D) Subcutaneous bleeding
E) Epidural bleeding
See explanation of question 15
(Answer A)
27. What is the cause of bloody vaginal discharge in a three-day-old baby? (September 89)
A) Hemolytic anemia B) Vitamin K deficiency
C) Vaginal adenosis D) Bleeding tendency
E) Maternal hormone
Female infants may have vaginal discharge or bleeding due to the hormones passed from the mother.
pathologic
It is not. Estrogen is withdrawal bleeding.
(Answer E)
28. When does the Moro reflex disappear when awake? (September 88)
A) 1 month B) 3 months C) 6 months D) 12 months E) Not lost
See explanation of question 23
(Answer B)
22
ALL TYPES OF TISTIME QUESTIONS

file:///C|/Users/User/Downloads/mcq.htm[04-Aug-19 10:20:40 AM]


NEWBORN
PREMATURITY, POSTMATURITY, SGA BABIES
one. In the neonatal period jaundice, which is directly bilirubin
does not depend on the increase? (May 2011)
A) Crigler-Najjar syndrome D) Tyrosinemia
C) Sepsis E) Cystic fibrosis
E) Ball atresia
Direct hyperbilurubinemia in the newborn period presents with cholestasis. There may be intrahepatic and
extrahepatic causes. Biliary atresia, metabolic causes, neonatal hepatitis, Allagille send-
rum, Caroli's disease, Byler's disease, alpha-1-antitrypsin deficiency, sepsis, cystic fibrosis, urinary tract
infections, drugs, hypothyroidism, Dubin-Johnson syndrome, Rotor syndrome, etc. why could it be. Wil-
does not affect the liver in the last newborn period, does not do cholestasis. Crigler-Najjar is a non-
cholestatic, non-hemaolytic bilurubin metabolism disorder with a high indirect bilirubin. Type-1 and
type-II have. Type-1 does not have the enzyme glucronil transferase, resulting in kernicterus. Type-II is
also high in indirect bilurubin. The use of phenobarbital in type II cases reduces jaundice.
(Answer A)
2nd. A healthy baby weighing 3600 grams had a Apgar score of 8 in the 5th minute and 9 in the 10th
minute.
Features. Jaundice is detected in the 12th hour of life. The blood type of the mother of the baby whose
blood group is A Rh (+) is 0 Rh (+). Total serum bilirubin level was 9 mg / dL.
It is found to be present. What is the most appropriate approach for this baby? (May 2011)
A) Checking the bilirubin level after 4 hours by telling the mother to breastfeed frequently
B) Starting the phenobarbital treatment and checking the bilirubin level after 4 hours
C) Initiation of preparations for blood exchange
D) Beginning of phototherapy and checking the bilirubin level after 4 hours
E) Initiation of fluid therapy considering that the baby is not fed enough
It's a very simple question for those with clinical experience;
Reach.
Hyperbilirubinemia in the first 24 hours, maternal blood group O Rh (+), infant blood group A Rh (+), ABO
incompatibility between mother and baby is. It should always be considered pathological. Hourly increments
0.75
mg, 5 mg / dl increased daily, 0.5 mg / dl increase in the case of this child may need to change blood, but
in any case, first FT and bilirubin level should be checked first.
It is used for the induction of phenobarbital enzyme in Crigler Najjar type 2 and Gilbert syndrome.
(Answer D)
ALL TYPES OF TISTIME QUESTIONS
23
PEDIATRICS
3. Indirect hyperbilirubinemia is detected in a term newborn due to jaundice, which is not within
physiological limits. Which of the following is the etiology?
Is not included in the examinations to be done in order to determine? (April 2010)
A) Determination of mother-baby blood group
B) Determination of glucose-6-phosphate dehydrogenase level
C) Direct Coombs test
D) Apt test
E) Osmotic fragility test
Mother infant blood types should be performed to determine ABO and RH mismatch. Again for the same
reason
Direct Coombs test should be performed to detect antibodies on infant erythrocytes. osmotic
fragility test is performed for hereditary spherocytosis. It is also the cause of disorder and erythrocyte
morphology
is an autosomal dominant disease. Determination of glucose 6-phosphate dehydrogenase level (X-linked)
It is especially important in boys and as a result of hemolysis may cause jaundice. Apt test
is made in the diagnosis of swallowed blood syndrome in the newborn and the blood to the mother or the
baby? belonging to
is used to indicate that.
Ingested blood syndrome: Blood in the stool due to swallowed blood or nipple cracking at birth.

file:///C|/Users/User/Downloads/mcq.htm[04-Aug-19 10:20:40 AM]


Apt test is used to differentiate from GIS bleeding. Fetal hemoglobin is resistant to alkali
It is based. When mixed with NaOH -coffee color is the mother of the source, the pink color persists
indicates that the baby.
Characteristics of pathological jaundice:
one. The first day of jaundice
2nd. 5 mg / dl increase per day
3. 12 in term and 14 mg / dl in preterm
4. Jaundice lasting more than 14 days
5. Direct bilirubin increase above 2 mg / dl.
6. History of hemolytic disease, pallor, hepatosplenomegaly, dehydration, excessive weight loss,
vomiting, kernicterus findings, acolic stool, darkening of urine color and pathological jaundice
It suggests.
In this case, the etiology should be examined.
Indications and indications to be requested in case of jaundice in infants
prepared table question.
24
ALL TYPES OF TISTIME QUESTIONS
NEWBORN
Protective factors for jaundice
• Total bilurib in low risk zone before discharge
• Discharge after the first 72 hours
• GH> 41
• Bottle feeding
• Black race
Major risk factors
• Total bilurib in high zone before discharge
• Jaundice in the first 24 hours
• GH 35-36
• Brother FT history
• Breastfeeding and weight loss
• Minor risk factors in Asian race
• Total bilurib high-middle zone before discharge
• Observation of jaundice during discharge
• GH 37-38
• Brotherhood jaundice
• Macrosomic baby, DAÇ
• Maternal age> 25
• Male gender
Indication Review
Jaundice for the first 24 hours TSB / TcB
Extremely high bilirubin TSB / TcB for age
FT taking baby or jumping percentile
Blood groups, Coombs test, complete blood count,
hyperbilirubinemia, history or physical examination
smear, direct bilirubin measurement, recurrent TSB unexplained jaundice
Bilirubin level at the blood exchange limit / FT
lack of response
25 G6PDH, reticulocyte count, albumin level,
Direct hyperbilirubinemia in urine assay, urine culture and sepsis
ratings
> 3 weeks jaundice / jaundice in sick infants
Evaluate TB and DB measurement and cholestasis if IB is high. Screening for thyroid and galactosemia
search for signs of potyroidism
(Answer D)
4. Which of the following oligohydramnios in newborn babies
is one of the complications? (April 2010)
A) Anencephaly

file:///C|/Users/User/Downloads/mcq.htm[04-Aug-19 10:20:40 AM]


B) Tracheoesophageal fistula
C) Pulmonary hypoplasia
D) Diabetes mellitus
E) Spinabifida
ALL TYPES OF TISTIME QUESTIONS
PEDIATRICS
Similar questions have been asked in previous years, which is the cause of oligohydramniosis / or
polyhydramniosis. The pathology which may be the cause and the result of both pulmonary hypoplasia-
d. The most serious complication is pulmonary hypoplasia.
Oligohydroamniosis Polyhydramniosis
Placental insufficiency, Anencephaly-hydrocephalus Preeclampsia GIS: Esophageal atresia, duodenal atresia,
TEF, cleft palate
and lip, IUGG AC: Cystic adenomatoid malformation, diaphragmatic hernia,
chylothorax Fetal anomalies Achondroplasia, Spina bifida, Werding Hoffman, trisomy 18,
21, Klippel-Feil Twin-twin transfusion (donor) Diabetic mother infant, Backwith-Wiedemann s Amniotic fluid
leakage Fetal anemia, fetal heart failure, twin-twin transfusion (recipient)
, hydrops fetalis, TORCH, Renal agenesis Polyuric renal disease
Urethral atresia Teratoma
Prune-belly syndrome
Pulmonary hypoplasia
Amniotic nodosum
indomethacin
ACE inhibitors
Intestinal pseudo obstruction
(Answer C)
5. In the evaluation of a baby born thirty-seven weeks according to the development charts as soon as it is
born; body weight is less than 3%, head environment-
is determined to be between 25-50%. Which of the following is most likely to occur in this baby? (April
2007)
A) Hypothermia, hypoglycemia and polycythemia
B) Respiratory distress syndrome
C) Increased incidence of group B streptococcal infection
D) Hypernatremia, hypokalemia
E) Although the weight is low, no problems are expected since the head circumference is within normal
limits.
The question asks the problems of SGA babies.
Problems and pathogenesis of SGA infants: Fetal death: Acidosis, hypoxia, infection, congenital anomalies.
Perinatal asphyxia: Chronic fetal hypoxia-acidosis, meconium aspiration, uteroplasental perfusion
reduction.
Hypoglycemia: Low depot, reduced glyconeogenesis, increased need (hypoxia, hypothermia and
relatively large brain tissue)
Dysmorphic features: Syndromes, genetic-chromosomal abnormalities, oligohydramnios-related anomalies,
TORCH infection. Symmetrical SGAs are more common.
26
ALL TYPES OF TISTIME QUESTIONS
NEWBORN
Polycythemia / hyperviscosity: Erythropoietin increase
Reduced O2 consumption / hypothermia: Hypoxia, hypoglycemia, fasting effect and lack of adipose tissue
Lung bleeding and persistent fetal circulation. Hypopotasemia and hypercalcemia are not the problem of
SGAs. If magnesium is given to the mother
hypermagnesemia. More common in premature: RDS, Apnea, IVK and neonatal death.
More common in SGAs: fetal death, meconium aspiration, air leak, persistent fetal circulation,
malformations, polycythemia, TORCH infections, hypoglycemia, polycythemia, congenital malformations,
persistent fetal circulation.
Although intrauterine growth retardation (IUGR) and SGA are used interchangeably, they are not
synonymous. IUGR refers to the inability to achieve optimal intrauterine growth. Intrauterine growth
Since insulin is the most effective factor, insulin deficiency, IGF -1 deficiency and pancreatic hypoplasia also

file:///C|/Users/User/Downloads/mcq.htm[04-Aug-19 10:20:40 AM]


makes IUBG.
SGA: Weight is below 10th percentile compared to gestational week. The most common cause of IUGR and
SGA is uteroplacental insufficiency.
(Answer A)
6. Which of the following is not expected in infants of preeclamptic mothers?
(April 2007)
A) Intrauterine growth retardation B) Polycythemia
C) Hypoglycemia D) Thrombocytopenia
E) Macrosomy
Placental insufficiency, preterm labor, SGA, asphyxia, Mg toxicity, neutro-
PEN, thrombocytopenia risk is high in these babies because of the high risk of SGA, polycythemia,
hypothermia and hypoglycemia conditions are also high
(Answer E)
7. Which of the following is not a problem for SGA 'babies? (September 2004)
A) Perinatal Asphyxia B) Hypoglycemia
C) Hypothermia D) Polycythemia
E) Hypopotasemia
Hypoglycemia and hypothermia are the major metabolic problems in SGA infants. Hypercalcemia and
hypopotasemia
It is not visible. Hypopotasemia is not seen in preterm infants. These include renal tubular acidosis,
glycosuria and hyperpotasemia due to impaired renal tubular function. Giving Mg to the mother
In the case of Mg may be height.
(Answer E)
8. Which of the following would be expected in infants with intrauterine growth retardation?
isn't one of the runs? (September 2002)
A) Polysystem B) Hypoglycemia
C) Hypercalcemia D) Intrauterine infection
E) Hypothermia
ALL TYPES OF TISTIME QUESTIONS
27
PEDIATRICS
See 5. Description of the problem.
(Answer C)
Increased risks in adulthood in premature and low birth weight infants:
• Obesity
• Type 2 DM
• Ischemic heart disease and hypertension.
9. Which of the following is proportional to height, weight and head circumference intrauterine
is not one of the factors that cause growth retardation? (April 2002)
A) Congenital infection B) Chromosomal abnormalities
C) Teratogens D) Fetal alcohol syndrome
E) Fetal malnutrition
Symmetrical SGA: Occurs in the early stages of pregnancy (<16 GH). The number of cells is small.
Postnatal
period can not catch his peers. The weight, height and circumference of the head remain proportional.
Causes: Intrauterine infections (heaviest in CMV and rubella), chromosomal abnormalities, radiation,
genetics, teratogens, severe maternal hypertension and congenital anomalies. Asymmetric SGA: The
relative protection of height and head circumference, lower weight.
Fetal malnutrition develops due to an event occurring in the late pregnancy. They catch their peers.
Causes: Placental insufficiency, maternal malnutrition, preeclampsia and chronic hyper-
blood pressure. Chronic heart AC, renovascular diseases, hemoglobinopathies and non-alcohol / drug use of
the mother
is also the cause of asymmetric SGA.
(Answer E)
10. Which of the following is not used to determine the neonatal age? (April 2001)
A) Nail growth B) Ear structure and cartilage
C) Nipple D) Lanugo

file:///C|/Users/User/Downloads/mcq.htm[04-Aug-19 10:20:40 AM]


E) Underfoot line
Ballard scoring using physical and neurological criteria for calculating gestational age
GH ± 2 weeks. It can be done in the first 1 week. New Ballard
has developed for low birth weight babies. Physical Maturity:
Skin structure, (pharyngitis, transparency, appearance of vessels ..) Lanugo (abundant in preterms, less in
term)
Soles of the soles (starts at the anterior, developing towards the heel)
Ear structure / eye Genital organs
Neuromuscular mature: posture, square window test (hand-wrist angle), arm movement (return to initial
position)
recoil), popliteal angle, heel-ear maneuver, scarf.
Although nails are developed in term and not developed in preterms, they are not one of the criteria used.
(Answer A)
28
ALL TYPES OF TISTIME QUESTIONS
NEWBORN
11th. Forty-first week of pregnancy; 2600 g born 48 cm and head circumference
Which of the following does a newborn 34 cm fit?
(April 2001)
A) Mature SGA doll
B) Premature SGA doll
C) Post mature SGA doll
D) Low birth weight baby
E) Mature normal weight baby
Low birth is independent of the weight of 2500 grams regardless of the week.
Very low birth weight: Birth weight less than 1500 grams
Extremely low birth weight: The birth weight is less than 1000 grams.
Term 37- 42 is used for babies born at gestational week.
Postterm identifies babies born at> 42 weeks.
SGA is below 10 p weight and LGA is above 90 percentile compared to gestational week.
Definitions.
Term baby SGA limit is 2500, LGA limit is 4000 gr.
Mean length at birth is 50 cm and head circumference is 35 cm.
(Answer E)
12. 1600 g of normal spontaneous delivery at the time of delivery
pathological finding which is most likely to be seen in the first 12 hours in a baby weighing
gut? (April 1999)
A) Anemia
B) Hypoglycemia
C) Cutis marmaratus
D) Hypotension
E) Hypercalcemia
Cutis marmaratus is a physiological condition caused by insufficient vasomotor control.Answer B)
13. Which of the following is not a sign of neonatal hypothyroidism? (September 94)
A) Intrauterine growth retardation
B) Large fontanelle and open fontanelle
C) Prolonged jaundice
D) Umbilical hernia
E) Large language
In hypothyroidism, babies are born as postmature and LGA. Prolonged jaundice, umbilical hernia and
fontanel-
Other findings are large.
(Answer A)
ALL TYPES OF TISTIME QUESTIONS
29
PEDIATRICS
14. Which of the following is the most common with postmaturity? (April 1990)

file:///C|/Users/User/Downloads/mcq.htm[04-Aug-19 10:20:40 AM]


A) Anencephaly
B) Renal agenesis
C) Urinary tract infection in mother
D) Polycystic kidney disease
E) Polyhydramniosis
POSTMATURITY: Pregnancy period exceeds 294 days or 42 weeks according to the last menstrual period.
Maturity.
Skin color pale, slight cyanosis of hands and feet, shortage of vernix casee, long nails, abundant hair,
Similar to parchment paper or desquamized skin, increased irritability is seen.
If there is placental insufficiency, meconium staining and subcutaneous fat tissue decrease.
Problems: Fetal distress, meconium aspiration, polycythemia, hypoglycemia.
Postmaturality: Anencephaly (the most common cause), trisomy 18, Seckel syndrome.
(Answer A)
15. Which of the following is wrong with premature babies? (April 1990)
A) Subcutaneous fat is less
B) Respiratory is rapid and the chest collapses
C) Breast tissue not developed
D) Foot soles formed
E) Labium major covers the minor
Physical properties of preterm babies
There is a physiological hypotonia in preterm infants.
The ratio of the head to the trunk is greater than that of a normal newborn (preterm megascephaly)
The fontanel is wide, the chest wall is soft and the abdomen is taut.
The skin is thin, gelatinous in appearance and covered with vernix caseous.
Subcutaneous adipose tissue is low. The body surface is wide relative to the scale. Consequently, heat loss
in preterms
And more.
Edema is usually an accompanying symptom of preterm labor.
Genital organs are less developed. In the boy, the testes did not descend to the scrotum and in the girl the
labium majorler minor-
s did not cover.
The structure of the ear cartilage is soft and the number of folds is low.
Although 0.75-1 cm in nipple term, it is not palpated or 0.5 cm in preterms
is less than.
The soles of the soles are not developed.
(Answer E)
30
ALL TYPES OF TISTIME QUESTIONS
NEWBORN
16. Which of the following is not a cause of prematurity? (September September 89, 87)
A) Diabetic mother
B) Smoking
C) Polyhydramniosis
D) Multiple pregnancy
E) Uterine anomalies
Smoking causes SGA. Preterm is not the cause of birth.
Preterm is not a cause of birth, SGA is not a cause of birth. Remembering that questions are likely to be
asked frequently as postterm birth is not the cause
It'll be helpful. When asked about the cause of preterm birth, SGA or postterm can be done more easily by
looking for causes of birth.
PRETERM CAUSES OF BIRTH
one. Fetal: Fetal distress, multiple pregnancy, erythroblastosis, non-immune hydrops
2nd. Plesental: Plesenta previa, abruptio plesenta, placental dysfunction 3. Uterine: Bicornate uterus,
cervical insufficiency (premature dilatation)
4. Maternal: Preeclampsia, infection (Listeria, GBS, urinary tract infection, chorioamnionitis, bacterial
vaginosis) chronic disease (cyanotic heart disease, renal disease), drug use
KA). 5. Other: Polyhydramniosis, early membrane rupture, trauma, iatrogenic

file:///C|/Users/User/Downloads/mcq.htm[04-Aug-19 10:20:40 AM]


Postmaturality: Anencephaly (the most common cause), trisomy 18, Seckel syndrome. Causes of SGA birth
• Fetal
• Chromosomal abnormalities (trisomies), congenital syndromes
• Infection (cytomegalic inclusion disease, rubella, syphilis)
• Multiple pregnancy
• Radiation
• Pancreatic hypoplasia, insulin deficiency, IGF 1 deficiency
• Placental causes
• Reduced placenta size, surface area
• Villous placentitis, infarction, placenta separation
• Tumors (koryoangioma, hydatiform mole)
• Twin-twin transfusion (donor)
• Maternal
• Toxemia
• Hypertension, renal disease
• Hypoxia (cyanotic heart, lung disease, living at height)
• Chronic disease, sickle cell anemia
• Drugs: Cocaine, antimetabolites, alcohol, cigarettes, narcotics
(Answer B)
ALL TYPES OF TISTIME QUESTIONS
31
PEDIATRICS
RESPIRATORY SYSTEM DISEASES
one. Newborn in babies delivered by cesarean section without labor
Which of the following is more common during the period? (September 2011)
A) Pneumonia
B) Hypoxic encephalopathy
C) Hypoglycemia
D) Temporary tachypnea of the newborn
E) Heart failure
One of the most important problems related to caesarean section is formerly known as lung or type II RDS.
takip Temporary tachypnea of the newborn (TTN) an. Even the most important risk factor for TTN is
cesarean delivery. Caesarean section with pneumonia, hypoglycemia, heart failure and hypoxic ischemic
encephalopathy
There is no direct relationship between birth.
TEMPORARY FOLLOW-UP OF NEWBORN It occurs due to the failure of fetal lung fluid to be cleared from
the lungs at birth. from AC
cleaning the liquid; catecholamine, vasopressin, prolactin, glycocorticoid increase and compression of the
thorax in the birth canal. The remaining part is cleaned by lymphatic drainage.
Term is common in infants born with borderline preterm, LGA and cesarean section. Risk factors elective
caesarean section (most importantly, especially in <38 GH), male sex, diabetic mother child, macrosomy,
excessive fluid to mother
and sedation, delayed cord clamping (ideal time 30-45 sec), polycythemia, breech presentation, asphyxia
and β2 exposure.
Tachypnea begins immediately after birth. Withdrawals and moaning. Cyanosis is rare and resolves with
minimal O2. The anterior-posterior diameter of the chest increases due to the increase in lift-off (barrel
chest). Resting lung
sounds are normal. Hypoxia, acidosis hypercapnia is rare. Usually heals rapidly within 3 days. Rarely, it is
severe (mostly in those born with elective caesarean section). Resistant hypoxia is seen and malignant
It is called TTN. It might take a week. Increased aeration on the X-ray, flattening of the diaphragm,
increased pulmonary vascularity, fluid in the fissures,
rarely pleural fluid. It is distinguished by clinical course and X-ray findings such as decreased aeration of
RDS, reticulogranular appearance and air bronchograms.
(Answer D)
32
ALL TYPES OF TISTIME QUESTIONS
NEWBORN

file:///C|/Users/User/Downloads/mcq.htm[04-Aug-19 10:20:40 AM]


2nd. Early neonatal period in a baby born 900 grams in the 27th week of pregnancy
respiratory distress syndrome developed and the baby 28 days mechanical ventilation and
received oxygen therapy. The baby is still in the incubator at 36 weeks postconceptional.
best oxygen requirement.
What is the most likely diagnosis for this baby? (September 2009)
A) Bronchopulmonary dysplasia
B) Pneumonia
C) Pulmonary hypertension
D) Temporary tachypnea of the newborn
E) Respiratory failure of premature
Bronchopulmonary dysplasia (BPD): Postconsep-
The presence of oxygen dependence on week 36 or day 28 is defined as BPD.
Classical BPD oxygen and mechanical ventilation (MV) in infants with RDS
is a chronic lung disease. Extrapulmonary causes (central apnea, diaphragm paralysis)
Infants receiving O2 and MV treatment do not develop unless lung parenchymal disease develops. New
BPD: a form seen in very low birth weight infants who did not need MV and O2 before. The development of
BPD is multifactorial. Atelectasis, ventilator-induced excessive aeration, free O2 radicals and
inflammation causes lung damage.
Protective factors: Vitamin A supplementation, early CPAP administration (most effective), caffeine. Antena-
does not protect tal steroids and surfactants.
Treatment: Keep calories high, fluid restriction, diuretics, β2 agonists and anticholinergics, methyl xanthines
are used. Dexamethasone cerebral palsy, neurological and somatic developmental defects
therefore, it is not used routinely. Inhaled beclomethasone reduces the need for systemic steroids and MV
separation
Easier.
Pulmonary vascular resistance is increased and there is abnormal vascular reactivity. Mild hypoxia even
pulmonary artery
causes an abnormal increase in pressure.
BPD cardiac complications: Pulmonary hypertension, cor pulmonale, systemic hypertension, left
ventricular hypertrophy, aortopulmonary collaterals.
The leading cause of death is due to cor pulmonale and RSV infections. Palivi- for RSV prophylaxis
zumab is held once a month during the month of November - March.
Risk factors for BPD
MV and O2 treatment due to RDS Interstitial emphysema
Prematurity and immaturity
Male gender
PDA, increase in pulmonary artery pressure
Giving excess fluid on the first day
Chorioamnionitis, genital Mycoplasma infection in mother
Family history of atopy / asthma
(Answer A)
ALL TYPES OF TISTIME QUESTIONS
33
PEDIATRICS
3. A baby born at 26 weeks of age with a diagnosis of respiratory distress syndrome
treatment to reduce the risk of developing bronchopulmonary dysplasia
Which is? (April 2009)
A) Surfactant treatment
B) Antibiotic treatment
C) Indomethacin treatment
D) Vitamin A treatment
E) Positive inotropic treatment
See 2. Description of the problem
(Answer A)
4. Which of the following is not a risk factor for sudden infant death syndrome (SIDS)?
(April 2009)
A) Baby's use pacifier

file:///C|/Users/User/Downloads/mcq.htm[04-Aug-19 10:20:40 AM]


B) Smoking of the mother
C) Laying the baby upside down
D) Baby sleeping in the same bed as the mother
E) Prematurity
While the use of a pacifier is not recommended beforehand, it can be used after 1 month after getting used
to the breast.
It is indicated.
Sudden Infant Death Syndrome (SDS)
SIDS syndrome; There was no feature in her anamnesis, including postmortem examination including
autopsy.
It is defined as sudden and unexpected infant death which cannot be determined. postnatal
The most common cause of mortality is 1 month -1 years. Most often occurs in 2-4 months. Genetic factors
in etiology
(cardiac Na / K channel, IL 10 gene polymorphism, complement components, serotonin gene polymorphism
and environmental factors are important.
The most important risk factor is the mother's smoking. Decreased frequency of sudden infant death in
supine position
It is about sleeping. In sudden unexpected infant deaths, SIDS and deliberate strangulation cannot be
separated by autopsy. Congenital anomalies,
infections can be divided into traumatic child abuse. Clearly life-threatening events and a history of SIDS in
siblings are also increased risk factors.
Vaccination is not a risk factor for SIDS. SIDS cannot be prevented by any screening method and
monitoring.
The risk of SIDS can be reduced by taking measures to reduce risk factors. After getting used to the nipple,
use the pacifier, baby sleeping on the back, separate beds in the same room
sleep, the baby's bed does not have soft covers, avoid overheating, such as the mother does not smoke.
34
ALL TYPES OF TISTIME QUESTIONS
NEWBORN
Environmental risk factors for SIDS Maternal and antenatal: AFP elevation, IUGR, alcohol, smoking drug
use, young uneducated and single
mother, malnutrition Child related: 2-4 months, male sex, prematurity, low birth weight, growth retardation,
prone or
side sleeping, no pacifier use, thermal stress, extreme cold, history of previous febrile illness
(Answer A)
5. Treatment of a premature baby with respiratory distress syndrome
which of the following does not apply? (September 2008)
A) Sodium bicarbonate treatment
B) Surfactant treatment
C) Positive pressure mechanical ventilation
D) Inotropic drug (dopamine) treatment
E) Betamethasone treatment
RESPIRATORY DISTRESS SYNDROME (RDS) (hyaline membrane disease) It occurs as a result of surfactant
deficiency. Diffuse atelectasis occurs. It is the disease of premature babies.
Lung appears dark purplish red, liver consistency. Alveoli are covered with acidophilic (hyaline membrane)
membranes due to plasma leakage. The most important risk for RDS is prematurity. insulin
delay the production of surfactants by antagonizing the effect of glucocorticoids. Clinical Findings: RDS
symptoms usually begin within minutes after birth. Tachypnea, moaning,
withdrawals are seen in cyanosis and increasingly missed and partially O2. Late onset tachypnea should
suggest other conditions. Increased dyspnea and cyanosis are typical in untreated RDS. be-
Symptoms and symptoms peak at 3 days, followed by gradual improvement. Spontaneous diuresis
and the reduction of O2 requirement is predictive of recovery. Death is usually 2-7. days, alveolar air
leakage occurs due to pulmonary or intraventricular. Prevention: If the lecithin / sphingomyelin ratio: 2/1 or
more, the risk of RDS is low. Diabetic mother
Phosphotidyl glycerol level is more reliable because it increases later in infants. Prenatal corticosteroid
(betamethasone) was given to the mother at 24-34 weeks of gestation, 48 hours before birth.
It is given. Reduces RDS and related complications. Perinatal dexamethasone should not be used.

file:///C|/Users/User/Downloads/mcq.htm[04-Aug-19 10:20:40 AM]


Periventricular leukomalacia develops, IVK, RDS and mortality are higher than betamethasone.
The aim of treatment is to provide oxygenation. Mechanical ventilation is applied. Liquid restriction is made.
Since GBS cannot be separated from pneumonia, ampicillin and gentamicin are initiated. Recurrent
Surfactant
The application can be done (2-4) times. If necessary, inotropic support or HCO3 may be used.
ECMO and inhaled NO are used for unresponsive treatment.
Surfactant use: Intratracheal. Immediately after birth (prophylactic) or in the first hours
The use of surfactant (early recovery) reduces air leakage and death from RDS. But
BPD does not change.
Complications from the sufra; Transient hypoxia, bradycardia, tube obstruction, pulmonary bleeding.
Betamethasone (not dexamethasone) is administered prophylactically to the mother.
(Answer E)
ALL TYPES OF TISTIME QUESTIONS
35
PEDIATRICS
6. In the first hour of a baby born 2800 grams by cesarean section in the 37th week of pregnancy
tachypnea and intercostal retractions are developing. Pulmonary vein
In the fissures, there is a slight appearance and fluid appearance is observed. Baby only
Oxygen treatment at the end of the 2nd day is healing.
What is the most likely diagnosis for this baby? (September 2007)
A) Hyaline membrane disease
B) Respiratory distress syndrome
C) Temporary tachypnea of the newborn
D) Meconium aspiration syndrome
E) Congenital pneumonia
Transient tachypnea of neonate (TTN): RDS type II, also called wet lung. Fetal lung
fluid cannot be cleared from the lungs at birth.
Term is common in infants born with borderline preterm, LGA and cesarean section.
Risk factors are elective caesarean section (most importantly, especially in <38 GH), male sex, diabetic
mother child,
macrosomy, excessive fluid and sedation to the mother, delayed cord clamping (ideal)
time 45 sec), polycythemia, breech presentation, asphyxia and β2 exposure.
Tachypnea begins immediately after birth. Withdrawals and moaning. Cyanosis is rare and minimal
Corrects with O2. The anterior-posterior diameter of the chest increases due to the increase in lift-off
(barrel chest). to relax
lung sounds are normal. Hypoxia, acidosis hypercapnia is rare. Usually within 3 days quickly
recover.
Rarely, it is severe (mostly in those born with elective caesarean section). Resistant hypoxia is seen and
malignant
It is called TTN.
Increased aeration on the X-ray, flattening of the diaphragm, increased pulmonary vascularity, fluid in the
fissures
and rarely pleural fluid. Decreased aeration in RDS, reticulogranular appearance and air
bronchograms.
(Answer C)
7. Which of the following causes respiratory distress syndrome in newborn infants
is not one of the cases? (September 2001)
A) Diabetes mellitus in mother
B) Prematurity
C) Perinatal asphyxia
D) Hypothermia
E) Induction history
The most important risk factor for RDS is preterm delivery. Increased risk of caesarean section while normal
birth
(with or without induction) reduces risk. Intrauterine stress increases cortisol release
increases the synthesis of surfactants. Insulin antagonizes the effect of steroids.
36

file:///C|/Users/User/Downloads/mcq.htm[04-Aug-19 10:20:40 AM]


ALL TYPES OF TISTIME QUESTIONS
NEWBORN
Increased Risk Decreased Risk
Prematurity Chronic Intrauterine Stress
Male Gender Female Gender
Cesarean section
Family history Prolonged membrane rupture time
Hypertension in asphyxia
Cold stress Cocaine / cigarette use
Diabetes in the mother IUBGG / SGA
Multiple pregnancy Steroid, thyroid hormones, theophylline, tocolytics
chorioamnionitis
(Answer E)
8. Born spontaneously vaginally at 38 weeks from a primiparous preeclamptic mother
Cyanosis is seen immediately in the baby, after ventilation with mask and oxygen
although the general condition of the baby is slightly better
It is observed that there is insufficiency, moaning and tachypnea. The most likely diagnosis in this baby
Which is? (September 2000)
A) Amniotic fluid aspiration
B) Wet lung
C) Hyaline membrane disease
D) Periventricular bleeding
E) Hypoglycemia
Preeclamptic mother, term infant and RDS, normal birth and clinical course cyanosis
temporary tachypnea (wet lung), periventricular hemorrhage due to term
It is departing.
In hypoglycemia and periventricular hemorrhage, there is no inadequate lung ventilation.
In this case, the most appropriate answer is aspiration of amniotic fluid. The first birth of the mother is
difficult birth
support and nasal aspiration of the mouth after birth is not specified
is another finding.
Amniotic fluid aspiration (fetal aspiration syndrome, aspiration pneumonia)
When prolonged and difficult delivery of placental O2 support decreases, the baby develops strong
respiratory movements. This
aspirate the amniotic fluid containing vernix, epithelium, blood, meconium and birth canal contents. Filling
alveoli and small airways may cause respiratory distress and, if infected, pneumonia
causes. Coarse granular appearance and irregular aeration are typical in fetal aspiration syndrome.
In order to prevent aspirations, firstly mouth and then nose aspiration should be performed. Postnatal
aspiration tracheoesophageal fistula, esophageal atresia, gastroesophageal reflux, duodenal obstruction
Zion may also be due to improper feeding and depressing drugs.
(Answer A)
ALL TYPES OF TISTIME QUESTIONS
37
PEDIATRICS
9. A 30-week-old premature baby has developed apnea on day 3. Which of the following is not used in the
diagnosis and treatment of the patient? (September 2000)
A) Apnea monitor
B) Theophylline
C) Hypoglycemia, infection intracranial hemorrhage tests
D) CBC, arterial blood gas, glucose, electrolyte tests
E) Start of antibiotics
APNEA: Breathing stops for 20 seconds or any time accompanied by bradycardia.
a stop.
Apnea serious illness seen after the first day or 14 days in preterm and at any time in term
The findings.
Idiopathic premature apnea: Depends on the immaturity of the respiratory centers. Between two-7 days
It happens. It is inversely proportional to the gestation week.

file:///C|/Users/User/Downloads/mcq.htm[04-Aug-19 10:20:40 AM]


Mixed apnea (50-75%) Obstructive apnea
Central apnea
Periodic respiration seen in preterm 5-10 sec. apnea period, rapid ventilation lasting 10-15 seconds
(50-60 / min) period follows. It is not pathological.
Convulsive apnea: It can be seen in apneic convulsions in the newborn. During this time, the heart rate
increase and
The presence of mild seizure symptoms (such as blinking eyes, slipping eyes) is important in differentiation.

Risky babies are monitored in the treatment of apnea. Antibiotic in idiopathic prematurity apnea after
exclusion of underlying metabolic disorders, anemia, infection, bleeding
there is no need to start. Apnea Treatment: Tactile stimulation is sufficient in mild apneas. In severe apnea;
airway opening, aspiration,
ventilation with oxygen and positive pressure; and, if necessary, intubation. In recurrent apneas:
Methylxanthines (theophylline, caffeine, aminophylline) respiratory stimulation and diaphragm
to increase contractions. Therapeutic indexes are narrow. Caffeine is more effective in central apnea and
reduces the risk of BPD. Doxapram stimulates breathing through peripheral chemoreceptors but
Side effects are high. If there is an underlying cause such as anemia, it is corrected.
(Answer E)
38
ALL TYPES OF TISTIME QUESTIONS
NEWBORN
10. A baby born on time and weighing 3300 gr.
physical examination and X-ray findings of the patient with developing tachypnea and cyanosis.
and arterial blood phases PO2: 33, PCO2 44, pH: 7.2
which is considered? (September 1999)
A) Persistent pulmonary hypertension
B) Interstitial pulmonary fibrosis
C) Temporary tachypnea of the newborn
D) RDS
E) Viral pneumonia
PERSISTAN PULMONARY HYPERTENSION (PPHT, PERSISTAN FETAL CIRCULATION)
Term and postterm infants' disease.
Risk factors; asphyxia, meconium asipration syndrome, early onset sepsis-pneumonia, respiratory
distress syndrome, pulmonary hypoplasia (oligohydramniosis, diaphragmatic hernia, pleural effusion).
Hypoglycemia, polycythemia, abnormal venous return
Maternal use of NSAIDs and serotonin reuptake inhibitors.
Plasma arginine and NO levels are low. The reason for PPHT is postoperative low pulmonary pressure,
foramen ovale and ductus arteriosus.
Pressure drop PO2 increase, PCO2 decrease, pH increase, release of vasoactive substances.
Development of hypertrophy in the smooth muscle layer of pulmonary arterioles, polycythemia and
intrauterine hypoxia
abnormal pulmonary venous return develops PPHT due to obstruction of pulmonary flow.
Hypoxia and cyanosis are incompatible with pulmonary findings. Hypoxia is labile. PCO2 normal or
slightly increased. Hypoxia is always found.
Systolic murmur due to tricuspid regurgitation, Hard and single S2 is heard. Preductal (right radial) and
postductal
(umbilical artery, lower extremity) PaO2 difference> 20 mmHg, saturation O2 difference> 5 right-left
shunt. Diagnosis is confirmed by ECO. The degree of tricuspid insufficiency increases pulmonary artery
pressure.
estimation.
General precautions: Baby from all intervention and external stimuli (blood collection, aspiration, sound-
light, etc.)
It must be protected.
Metabolic disorders, hypothermia and polycythemia are corrected.
For pulmonary vasodilation, O2 is increased with MV and alkalosis with NaHCO3 is tried.
Hypocarbia and alkalosis can cause neurosensory hearing loss and cerebral palsy.
It should be monitored.

file:///C|/Users/User/Downloads/mcq.htm[04-Aug-19 10:20:40 AM]


Inotropic agents (dopamine) are used to increase systemic pressure.
Thalazoline is a nonselective α blocker.
Inhaled NO is used as a pulmonary vasodilator, its side effect is methemoglobinemia. Sildenafil fos-
phodiesterase 5 inhibitor. Increases the effect of NO by reducing degradation of CGMP. Prostoglandin I2
and MgSO4 are also used as vasodilators. ECMO is applied to patients who do not respond to treatment.
Heparin
There is a risk of bleeding and using <2 kg, <34 weeks of gestation.
(Answer A)
ALL TYPES OF TISTIME QUESTIONS
39
PEDIATRICS
11th. If the newborn baby is cyanotic while sucking and cyanosis regresses while crying,
Considered? (September 98)
A) Koanal atresia
B) Tetralogy of Fallot
C) VSD
D) Persistent pulmonary hypertension
E) Tricuspid atresia
Koanal atresia: It is the most common congenital anomaly of the nose. The recovery of cyanosis when
crying is typical. Most of them are due to bilateral and bone structure. Congenital anomalies accompanying
half of them
has. It is accompanied by CHARGE association. Diagnosis is made when the nasogastric catheter is not
advanced. The best method for evaluating the anatomic structure is high resolution CT.
Common life-threatening anomalies at birth
Koanal atresia
40 ALL TUSH QUESTIONS TO TREATMENT Postpartum respiratory distress, apnea, failure of catheter to
pass through nostril, accompany CHARGE association
Pierre Robin syndrome Micrognathia, cleft palate, airway due to glossoptosis
Obstruction
Tracheesophageal fistula / esophageal atresia
Polyhydramniosis, excessive salivation, aspiration pneumonia, inability to advance the catheter into the
stomach, accompanies VATER association
Diaphragmatic hernia Scaphoid abdomen, respiratory distress, bowel sounds of the thoracic
receipt of sta
Intestinal obstruction: duodenal atresia, volvulus, intestinal atresia
Related to polyhydramniosis, biliary vomiting, abdominal distention, trisomy 21, cystic fibrosis and cocaine
use
Gastrochisis / omphalocele Polyhydramniosis, intestinal obstruction findings
Neural tube defect: anencephaly, meningomyelocele AFP elevation, polyhydramniosis, decreased fetal
movement
Renal agenesis, Potter syndrome Oligohydramniosis, pulmonary hypoplasia, pneumothorax
Ductus-dependent congenital heart diseases Cyanosis, hypotension, murmur
(Answer A)
12. Born after forty-two weeks of pregnancy and amniotic fluid is meconium
What is the first intervention if a resuscitation is indicated in a newborn? (April 98)
A) Upper respiratory tract aspiration
B) Endotracheal intubation
C) Ventilation with air sac and mask
D) Cardiac compression
E) Drug administration
NEWBORN
MECONIUM ASPIRATION SYNDROME (MAS): In 5-15% of births, amniotic fluid
Although onion, meconium aspiration syndrome develops in 5%.
Term and postterm infants' disease. Thirty-four GH is rare before.
Risk factors: Postterm pregnancy (most important), preeclampsia-eclampsia, hypertension in the mother,
cardio-
vascular disease, such as intrauterine growth retardation.

file:///C|/Users/User/Downloads/mcq.htm[04-Aug-19 10:20:40 AM]


Meconium amniotic fluid is usually associated with fetal hypoxia. Most aspirations during delivery
It happens.
Meconium amniotic fluid: mouth-nose and pharynx are aspirated as soon as the head comes out. If the
baby is active
(good breathing, KTA> 100, tonus good) routine maintenance. If the baby is depressed, intubated
tracheal aspiration is performed. If meconium is detected under the vocal cords, repeated aspiration-
s should be done. When MAS develops, its treatment is supportive. Surfactant is given.
Clinical Findings: Postmature symptoms, meconium staining (detected on the first cord) are detected. Light
The symptoms vary from respiratory distress to respiratory failure.
On examination, tachypnea, withdrawals, AP diameter increase, cyanosis, auscultation ral, rocus and
wheezing may be heard.
Atelectasis and ventilation perfusion disorder as a result of complete obstruction of the airways with
meconium,
partial confinement and air trapping - air leaks are seen.
MAS also causes chemical pneumonia and surfactant inactivation. All these factors are hypoxia, acidosis,
hypercapnia causes pulmonary hypertension. This is the ductus arteriosus and foramen
oval from right to left shunt and increases hypoxia.
X-ray patchy coarse granular infiltration, hyperinflammation, flattening of the diaphragm and
Anterior posterior diameter increase is detected.
Surfactants used in term infants: MAS, pneumonia, surfactant protein B, C deficiency, con
genital diaphragmatic hernia.
(Answer A)
13. In a baby whose cyanosis disappears after crying and lungs found normal by listening
Which of the following procedures should be performed for diagnosis? (April 96)
A) ECG is taken
B) Naloxane is given
C) Nasogastric catheter insertion into the nose
D) Chest X-ray is taken
E) Abdominal USG
Opening cyanosis when crying is typical for coanal atresia. Naloxane is an opioid antagonist and
It is used to correct depression caused by opiods given.
(Answer C)
ALL TYPES OF TISTIME QUESTIONS
41
PEDIATRICS
14. A child born in 1700 grams of thirty-two weeks had periodic apneas lasting 20-25 seconds. What do
you think of this kid?
(September 95)
A) Immature respiratory center
B) Periodic respiration
C) Hypoglycemia
D) Sepsis
E) Hypocalcemia
(Answer A)
15. What is the purpose of neonatal resuscitation? (April 94)
A) Inhalation
B) To provide central nervous system oxygenation
C) Increasing heart rate
D) Opening cyanosis
E) Correcting tone
The aim is to maintain oxygenation of the nervous system by providing adequate oxygenation and
ventilation.
(Answer B)
16. With a history of polyhydramniosis in the mother
What is the diagnosis of the baby in which the secretion is detected? (September 88)
A) Congenital pyloric stenosis
B) Aganglionic megacolon

file:///C|/Users/User/Downloads/mcq.htm[04-Aug-19 10:20:40 AM]


C) Esophageal atresia
D) Meckel's diverticulum
E) Invagination
Although it is not very relevant, it is among the life-threatening anomalies.
(Answer C)
42
ALL TYPES OF TISTIME QUESTIONS
NEWBORN
GASTROINTESTINAL SYSTEM PROBLEMS
one. Which of the following is necrotizing enterocolitis in premature newborns?
is the definitive diagnostic finding for? (September 2009)
A) Abdominal distension
B) Pneumatosis intestinalis on direct abdominal X-ray
C) Determination of acidosis in blood gases
D) Vomiting
E) Detection of thrombocytopenia in whole blood count
NECROTIZING ENTEROCOLITIS (NEC): NEC is the most common GIS emergency. Mucosal or transmural
involvement
It makes. It most commonly involves the terminal ileum and proximal colon. Most often in the first 2 weeks
of life.
Start time is inversely proportional to gestational age. Up to 3 months in very low birth weights
visible.
Etiology: Intestinal ischemia, enteral nutrition and pathogenic microorganisms.
Prematurity is the major risk factor. Related microorganisms; E. coli, Klebsiella, C. perfiringens, S.
epidermidis, rotavirus.
In preterm infants, it starts after feeding with formula. Enteral nutrition
before the onset and in breast-fed.
The first clinical signs of GIS are abdominal distension and gastric retention, and the first systemic finding is
lethargy.
and heat irregularity. Perforation of fecal occult blood positivity (recognized by guaiac acid test),
peritonitis and shock. The first thrombocytosis from laboratory findings
tope polymers. As the disease progresses, signs of neutropenia, acidosis and DIC develop.
Pneumomatosis intestinalis on the X-ray is diagnostic. Portal system
Having air (USG shows better) indicates advanced disease. Pneumoperitoneum can be detected.
In the treatment, feeding is stopped and nasogastric drainage is performed. Infection therapy and
parenteral nutrition
Applied.
Absolute surgical indications: Perforation, positive paracentesis (feces or microrganism).
Relative surgical indications: non-response to treatment, single fixed bowel loops, erythema of the intestinal
wall,
palpable mass.
Stress and short bowel syndrome are late complications.
Differential diagnosis: Systemic-intestinal specific infections, GIS obstruction, volvulus and isolated intestinal

perforation
(Answer B)
ALL TYPES OF TISTIME QUESTIONS
43
PEDIATRICS
2nd. Which is associated with the pathogenesis of necrotizing enterocolitis in the newborn
is not one of the risk factors? (April 2009)
A) Intestinal ischemia
B) Oral nutrition
C) Postmaturity
D) Pathogenic microorganisms
E) Low birth weight
The greatest risk factor for NEC is prematurity. Breastfeeding, parenteral nutrition and post-

file:///C|/Users/User/Downloads/mcq.htm[04-Aug-19 10:20:40 AM]


Maturity is not a risk.
(Answer C)
3. Which is one of the findings observed in necrotizing enterocolitis
It is not? (September 2006)
A) Abdominal distention
B) Bloody stool
C) Sepsis
D) Intestinal perforation
E) Alkalose
NEC is the cause of acidosis. See 1. Description of the problem
(Answer E)
4. Which of the following is the most common cause of cystic fibrosis in newborn babies?
Is the evidence? (April 2006)
A) Bronchopulmonary dysplasia
B) Necrotizing enterocolitis
C) Meconium ileus
D) Pneumatosis intestinalis
E) Rectal prolapse
The most common presentation of cystic fibrosis in children is recurrent airway findings in newborns.
The most common cause is meconium ileus.
MECONIUM ileus is the most common finding of cystic fibrosis in neonatal period. Pancreatic enzyme-
It causes obstruction in the terminal ileum due to the lack of smears and meconium being viscous.
Abdominal distention and vomiting are seen. Sister history of cystic fibrosis, abdominal obstruction
findings and meconium filling the intestines.
On the X-ray, a foamy granular image (lower right) is detected. Infants in terms of cystic fibrosis
It should be evaluated. Sweat test should be done. Meconium peritonitis and cystic fi-
broza is more specific.
Pneumotasis intestinalis is the presence of air in the intestinal wall. It is the radiological finding that makes
the diagnosis of NEC.
44
ALL TYPES OF TISTIME QUESTIONS
NEWBORN
MECONIUM PLUG: Obstruction of meconium in the lower colon or anorectal region.
Micro left colon (DAÇ), cystic fibrosis, aganglionic colon, magnesium administration to the mother and
opioid
with the habit. Glycerin is treated using isotonic gastrografin. After treatment
aganglionic megacolon.
(Answer C)
5. Which of the following should be considered in a patient with meconium ileus? (September
2004, September 97)
A) Hypothyroidism
B) Aganglionic megacolon
C) Cystic fibrosis
D) Alpha 1 antitrypsin deficiency
E) Wilson's disease
See the description of question 4.
(Answer C)
6. Which examination of a newborn with blood after 12 hours of meconium
It should be done? (April 99)
A) Gastric lavage
B) Direct abdominal radiography
C) OMD (esophageal, gastric duodenal radiograph)
D) Column graph with barium
E) Apt test
Is your apt test your mother? The baby? to understand that it belongs. Fetal hemoglobin alka-
resistant to high resistance. When mixed with NaOH yellow -coffee color is the mother of the source,
The continuation of the pink color indicates that the baby.

file:///C|/Users/User/Downloads/mcq.htm[04-Aug-19 10:20:40 AM]


Ingested blood syndrome: Blood in the feces is seen due to swallowed blood or nipple cracking at birth.
The menu. Apt test is used to differentiate from GIS bleeding.
Kleihauer-Betke (KB) test is used to determine the amount of fetal erythrocytes in maternal blood.
This test is based on the fact that erythrocytes are resistant to acid. Fetomaternal bleeding is detected and
Rh im-
anti-D globulin dose can be calculated for prophylaxis in munization.
(Answer E)
ALL TYPES OF TISTIME QUESTIONS
45
PEDIATRICS
7. Two thousand grams prematurely born, respiratory distressed child is given high-pressure oxygen
therapy. Food is started on the 4th day, tolerates the food, on the 9th day in the baby
abdominal distension develops, cyanosis begins, and bloody stinky stools. Intestinal obstruction, abdominal
wall and vena porta
If air shadows are seen, which is the possible diagnosis? (April 98)
A) NEK
B) Meconium ileus
C) Hypothyroidism
D) Aganglionic megacolon
E) Peritonitis
See 1. Description of the problem.
(Answer A)
8. Blood is detected in a stool in a newborn child. It is of mother or child origin
Which of the following is the test to determine whether the test? (April 95)
A) Apt test
B) Acid Raw test
C) Figlu test
D) Guiac test
E) Benedict test
The Benedict test is intended to detect reducing sugars.
Guaic acid test is used to detect occult blood in feces. Figlu test is used to show folic acid deficiency.
Formimus in urine after histamine administration
noglutamic acid increase is detected. Acid Raw test is used in paroxysmal nocturnal hemoglobinuria.
(Answer A)
9. Restlessness, nausea, vomiting, bloody 10 days after birth in newborn
diarrhea, abdominal distention occurs. Direct X-ray showed pneumatosis intestinal state. What do you think
of this patient? (April 95)
A) Ulcerative colitis
B) Hirschsprung's disease
C) Anal atresia
D) Necrotizing enterocolitis
E) Invagination
NEC is most common in preterm infants who are fed with formula during the first 2 weeks of life. See
question 1
description.
(Answer D)
46
ALL TYPES OF TISTIME QUESTIONS
NEWBORN
10. What is considered if there is gas in the vena port on the direct graph? (94 N)
A) Necrotizing enterocolitis
B) Liver abscess
C) Portal hypertension
D) Pancreatitis
E) Meconium peritonite
The presence of air in the Vena port is an indication of NEC's progress. Better than direct radiography with
USG

file:///C|/Users/User/Downloads/mcq.htm[04-Aug-19 10:20:40 AM]


It is determined. See description of question 1.
(Answer D)
11th. What is the radiological finding in a baby with meconium ileus? (September 89)
A) Enlargement of the heart contours
B) Innovation in costumes
C) Air shadow under aperture
D) Psoas shadow becoming clear
E) Foam appearance in the lower right quadrant
See 4. Description of the problem.
(Answer E)
ALL TYPES OF TISTIME QUESTIONS
47
PEDIATRICS
CENTRAL NERVOUS SYSTEM PROBLEMS
one. In premature infants, periventricular-intraventricular hemorrhage is most commonly
in which regions? (September 2008)
A) Subependimal germinal matrix
B) Capsula interna
C) Hippocampus
D) Insula
E) Substansia nigra
INTRACRANIAL HEMORRHAGE (ICC): It may occur in term and preterm infants.
Term trauma in infants is due to asphyxia and coagulation disorder. Subarachnoid, subdural
and subtentorial.
The ICC in the preterm is periventricular-intraventricular and subepandimal germinal matrix.
Caused.
Traumatic epidural, subdural and subarachnoid hemorrhages may occur when the head is larger than the
pelvis of the mother, prolonged births, breech presentation and device use.
Periventricular-intraventricular hemorrhage (PV-IVC): It occurs in premature. It occurs in the gelatinous
sub-epandymal germinal matrix. Embryonal neuron and fetal glial migrate to the cortex of this area
is the source of cells. It undergoes involution towards Terme and vascular maturation increases. Therefore,
periven- term babies
tricular bleeding is not seen. Predisposing factors for IVC: Prematurity (most important), RDS, hypoxia-
ischemic and hypotensive
damage, reperfusion, increased or decreased cerebral blood flow, decreased vascular integrity, increased
venous pressure, pneumothorax, hypervolemia, hypertension.
Most often occurs in 2-3 days. Late bleeding may occur in two to four weeks. It is rare after the first
month. USG is recommended for screening for 3-5 days under thirty-four weeks.
Clinical findings are variable. Asymptomatic or sudden drop in hemotocrit, apnea, bradycardia,
acidosis and seizures.
2-3 of life. sudden drop or transfusion of hematocrit in a premature patient whose general condition
Although not elevated, in case of jaundice should suggest IVC.
Unlike subdural and subarachnoid hemorrhage, USG can be used for the diagnosis and staging of IVC. The
treatment is followed in terms of supportive treatment and possible hydrocephalus. Shunt develops
hydrocephalus
or external drainage. Diuretic and acetozolamide are useless in treatment.
IVC poor prognostic indicators: Shunt requiring hydrocephalus, advanced stage bleeding, excessive PVL,
diffuse paran-
kimal bleeding.
Hydrocephalus, mental, motor retardation, hearing impairment and cerebral palsy after IVC (spastic in
preterm
diplegia).
48
ALL TYPES OF TISTIME QUESTIONS
NEWBORN
Periventricular cerebral infarction: It occurs due to venous congestion after severe IVK.
Periventricular leukomalacia (PVL): Periventricular white matter is focal necrosis or diffuse damage.

file:///C|/Users/User/Downloads/mcq.htm[04-Aug-19 10:20:40 AM]


Although it is associated with IVK, it is mostly due to ischemic damage, not bleeding. Accelerating PVL
formation: Fetal growth retardation, hypothyroxinemia, hypo / hypercarbia, fetal
vasculitis, maternal / placental infection. Possible protective factors from PVL: Antenatal corticosteroids,
prostoglandin inhibitors,
Magnesium decreases with the administration of antenatal steroids (betamethasone). Dexamethasone
periventricular leukomalacia
increases the risk. The administration of low-dose indomethacin is controversial.
2nd. Perinatal hypoxia and acidemia are directly related to the development of
is not responsible? (April 2003)
A) Neutropenia B) Persistent fetal circulation
C) Respiratory distress syndrome D) Cerebral edema
E) Disseminated intravascular coagulation
Hypoxia is the reduction of arterial oxygen and insufficient blood flow to ischemia. Perinatal asphyxia:
Defines hypoxia, hypercapnia and metabolic acidosis as a result of deterioration of gas exchange. Increased
anaerobic metabolism in fetal hypoxia with lactic acid and excitatory amino acids (especially gluta-
mat), free radicals and NO increases. Intracellular edema occurs with increased intracellular Na and Ca.
Blood flow is drawn from the lung, liver and intestine to the brain, heart and adrenals. Prognosis is
associated with the treatment of concomitant complications, gestational age and hypoxic ischemic
encephalopathy.
It varies according to the degree of (HIE). Poor prognosis: 1. Stage II-III HIE 2. Low APGAR at 20 minutes,
lack of spontaneous respiration and abnormal neurological findings at 2 weeks
presence of symptoms 3. Abnormal EEG and MRI findings. The use of EEG and MR in term infants is the
most effective method for predicting prognosis.Answer A)
Effects of asphyxia
• CNS: HIE, infarction, HRC, convulsion, cerebral edema, hypotonia, hypertonia
• Cardiovascular: Myocardial ischemia, contractility disorder, tricuspid regurgitation, hypotension.
• Pulmonary: Persistent fetal circulation, pulmonary bleeding, RDS
Renal: Oliguria, acute tubular or cortical necrosis, ARF.
• Adrenal: Adrenal bleeding
• Gastrointestinal: Paralytic ileus, NEC, perforation, necrosis, elevated liver enzymes, elevated ammonia and
bilirubin, decreased coagulation factors.
• Metabolic: Improper ADH secretion, hyponatremia, hypoglycemia, hypocalcemia, myoglobinuria
• Skin: Subcutaneous fat necrosis
• Hematologic: DIC, thrombocytopenia and increased erythrocytes with nuclei.
ALL TYPES OF TISTIME QUESTIONS
49
PEDIATRICS
3. What is not seen in perinatal hypoxia? (September 97)
A) DIC
B) Convulsion
C) Hyperglycemia
D) Heart failure
E) Hemolysis
See. 2 Questions.
(Answer C)
Symptoms of hypoxia in the fetus:
• IUGG
• Bradycardia
• Variability reduction in NST
• Late (uteroplacental insufficiency) or variable deselation (cord compression) pattern
• pH <7.2 in saline blood analysis
Symptoms of hypoxic ischemic encephalopathy according to stages in term infant
Parameter Stage 1 Stage 2 Stage 3
Consciousness Hyperalert Lethargic Stuppor / coma
Tonus Normal Hypoton Tonus too ↓
Posture Normal Flexion Deserebre
Reflex Hyperactive Hyperactive None

file:///C|/Users/User/Downloads/mcq.htm[04-Aug-19 10:20:40 AM]


Moro Strong Weak No
Pupil Mydriatic Myotic IR weak, anisocoria
Duration <24 hours 1- 14 days Weeks
4. Birth in a primiparous woman, vaginal delivery at term
After absorption, paralysis, hypotonia and pallor are detected. Soon the baby is lost. What is the possible
diagnosis? (September 92)
A) Medulla spinalis injury B) Pneumonia
C) Kernicterus D) Myelodysplasia
E) Amyotonia congenita
Spine and Spinal Cord Injury: It is usually difficult and occurs at birth. Most common arrival
C7-T1, C4 vertebrae are affected on head arrivals. Spinal cord rupture, bleeding and edema are rarely seen.
Clinical findings are related to the level of the lesion and
varies by weight. In the region under the lesion there is a loss of neurological function of type 1 neuron.
Heavy ones are lost immediately after birth. First day apnea with poor prognosis, 3
is the lack of motor recovery in the month. The differential diagnosis includes Amyotonia congenita,
myelodysplasia with occult spina bifida.
(Answer A)
50
ALL TYPES OF TISTIME QUESTIONS
NEWBORN
5. The newborn was born 8 hours before birth and 12 hours before forceps.
kika lasting convulsions and 1 minute apgar: 3, 5 minute apgar: 5 is
Learned. What is the diagnosis? (September 93)
A) Kernicterus B) Meningitis
C) Intracranial tumor D) Hypercalcemia E) Asphyxia
Difficult delivery with forceps, low APGAR and convulsion asphyxia on the first day. Blood group
mismatches are the most common cause of kernicterus. No such problem has been mentioned.
During the intrauterine period, the placenta clears indirect bilirubin. Therefore, it may be later. Symptoms
start in 2-5 days in term babies and in 7th day in preterm babies. Initial findings
decreased absorption, lethargy and loss of Moro reflex. Meningitis may cause seizures, but more often
accompanied by late sepsis.
Convulsion is not hypercalcemia but hypocalcemia and seizure with good prognosis. Intracranial tumors are
very rare in the newborn. The most common brain tumor under one age is choroidal plex-
choroid plexus papilloma or carcinoma. It causes hydrocephalus by causing CSF secretion.
(Answer E)
6. What is the most common cause of convulsions in the newborn? (April 92)
A) Hypocalcemia B) Birth trauma
C) Hypoglycemia D) Hyponatremia E) Hypomagnesemia
The most common cause is hypoxic ischemic encephalopathy. First day. As it may cause hypoxia,
The answer is birth trauma. Neonatal convulsions: branching of axons and dentrites in newborn period and
myelinization
In general, tonic-clonic seizures are not seen in the newborn period. The most common seizures are
amorphous (subtle) salivation, pedaling, apnea seizures. Newborn
seizures can also be confused with non-seizure tables. Tips for separating the seizure:
- There are autonomic changes such as tachycardia and increased blood pressure. - Sensory stimuli and
non-seizures are triggered, but seizures are not affected.
- Tables without seizures do not change when the baby stops when the seizure stops. Patients with seizures
and EEG discharge clinically; Focal clonic, focal tonic and some myoclonic seizures
It is seen. The seizure is epileptic and responds to antiepileptics. Clinical signs of seizure No EEG discharge:
Generalized tonic seizures. Often
infants with hyposic ischemic encephalopathy. Patients with non-clinical EEG discharge; It can be seen in
comatose babies without anticonvulsant.
Etiology and diagnosis: Hypoxic ischemic encephalopathy is the most common cause in neonates. In
addition, seizures due to metabolic, infectious, traumatic, structural, hemorrhagic, embolic and maternal
causes may occur. Newborn
Glucose, calcium, magnesium, electrolytes and BUN should be checked in infants who have seizures during
the period.

file:///C|/Users/User/Downloads/mcq.htm[04-Aug-19 10:20:40 AM]


LP and cranial imaging should be performed in seizures not related to metabolic causes. Traumatic PL:
Bloody CSF to be clear after centrifugation
Subarachnoid / intraventricular hemorrhage: CSF xanthochromic color.
(Answer B)
ALL TYPES OF TISTIME QUESTIONS
51
PEDIATRICS
METABOLIC DISORDERS AND DIABETIC MOTHER'S
one. Which of the following findings is not seen in children with diabetic mothers? (May 2011)
A) Hypoglycemia
B) Hypocalcemia
C) Tachyne
D) Tremor
E) Hypermagnesemia
Diabetic mothers are frequently asked and still popular. Its main metabolic problems are hypoglycemia,
hypocalcemia and hypomagnesemia.
Hypomagnesemia and hypermagnesis were used during the questioning. Most of the clinical manifestations
LGA-born polycythemic infants. All organs except the brain are large.
one. Hypoglycemia should be checked for blood sugar in the first hour. 2nd. Hypocalcemia and
hypomagnesemia
3. Polycythemia and jaundice III sheet page 509 Diabetic mothers: Congenital abnormalities are caused by
periconceptional glucose elevation. It is prevented by checking. Early
Acidosis due to the relative deficiency of insulin in the postoperative period leads to congenital anomalies. 3
times more than non-diabetics.
Findings due to fetal hyperglycemia:
one. Congenital anomalies
2nd. Decreased growth (0-20 GH, in early pregnancy) 3. Hyperinsulinemia (> 20 GH)
Findings due to fetal hyperinsulinism:
one. Hypoglycemia (0-7 days)
2nd. Lack of surfactant 3. Immature liver metabolism
Fetal hyperglycemia and hyperinsulinism-related findings
one. Fetal macrosomia (> 20 GH): labor asphyxia, transient tachypnea, cardiomyopathy
2nd. Fetal hypoxia (> 30 GH): Polycythemia, iron abnormalities Increased risk of renal vein thrombosis and
stroke due to polycythemia. Cardiomegaly occurs asymmetric septal hypertrophy and idiopathic subaortic
stenosis Digoxin and other positive inotropics cause worsening. RDS and tachypnea are more common.
(Answer E)
52
ALL TYPES OF TISTIME QUESTIONS
NEWBORN
2nd. Which of the following diseases may be associated with neonatal hypocalcemia?
(September 2009)
A) Down syndrome
B) Turner syndrome
C) DiGeorge syndrome
D) VATER syndrome
E) Apert syndrome
Dion syndrome is associated with neonatal hypocalcemia. Third and fourth pharyngeal arch
It is defect. It can be recognized by hypocalcemic convulsions in the neonatal period. Parathyroid hypopla-
Disorder, conotural cardiac anomalies and thymus hypoplasia can be seen. Causes T cell deficiency.
Increased tendency to infection increased with fungi, viruses and P. jirovecii (formerly P. carini).
Laboratory findings:
• Low PTH / c AMP low in urine
• Low calcium and phosphorus
• Low D, 1, 25 OH (PTH 1 alpha hydrocylase actives)
• The alkaline phosphate level may vary.
(Answer C)
3. Which of the typical findings of Beckwith-Wiedemann syndrome

file:///C|/Users/User/Downloads/mcq.htm[04-Aug-19 10:20:40 AM]


is not one? (September 2006)
A) Makroglossi D) Severe anemia
B) Omphalocele E) Microcephaly
C) Hepatoblastoma
Beckwith-Wiedemann syndrome (BWS), also known as omphalocele, macroglossia, gigantism syndrome.
So-
The reason for the incidence of metabolic disorders is hyperinsulinemia and hypoglycemia. Typical findings
of the syndrome: Macroglossia, omphalocele, macrosomy and ear lobe. The disease is related to 11
chromosomes and is caused by excessive release of insulin-like growth factors.
rosomy. Liver, kidney and pancreas also grow. It carries the risk of hemihypertrophy and increased
malignancy.
Increased malignancies:
• Wilms tumor (children with BWS should be followed by abdominal USG)
• Hepatoblastoma (alpha fetoprotein tracking)
• Adrenocortical tumors
• Gonodal tumors
• Increased insulin-induced polycythemia. No anemia.
• Points to take into consideration: History is polyhydramniosis, not oligo.
• It becomes an omphalocele, not gastrochisis.
• Increased tumor risk is Wilms, not neuroblastoma.
(Answer D)
ALL TYPES OF TISTIME QUESTIONS
53
PEDIATRICS
4. Which of the following is congenital, which can be seen in babies of diabetic mothers?
is not one of the anomalies? (September 2006)
A) Gastric duplication
B) Duodenal atresia
C) Cardiac malformations
D) Neural tube defect
E) Small left colon syndrome
Diabetic mothers are always asked about their babies. The last updated edition with why there are many
questions at the bottom
We're looking at.
Tricuspid atresia, which is one of the cardiac valve anomalies, was replaced with pulmonary and asked. In
Mitralle
It was changed.
Double outlet right ventricle, left ventricular hypoplasia. Aortic coarctation. Pulmonary artery
no poplasis. Or the most common anomalies asks which are not common pathologies
cardiac and lumbosacral agenesis.
On the contrary, there is asymmetric septal hypertrophy.
While there was no problem with the right column, the left column remained small.
Diabetic mother with her baby
Common congenital anomalies:
Cardiac malformations: VSD, ASD, transposition of the great artery, truncus arteriosus, double outlet
right ventricle, aortic coarctation and tricuspid atresia
Lumbosacral agenesis
Other accompanying anomalies:
Neural tube defect, Holoprosencephaly (Diabetes-related migration defect)
Renal agenesis, dysplasia, hydronephrosis, double ureter
(Answer A)
5. Which of the following is the most common cause of hypocalcemia in newborn babies?
not from the findings? (September 2006)
A) Convulsion
B) Vomiting
C) Lethargy
D) Carpopedal spasm

file:///C|/Users/User/Downloads/mcq.htm[04-Aug-19 10:20:40 AM]


E) Tremor
54
ALL TYPES OF TISTIME QUESTIONS
NEWBORN
Hypocalcemic neonates do not have carpopedal spasms. In neonates, ir-
ritability, jitterines and tremor. It may also cause sepsis-like findings in newborns.
can.
Hypocalcemia total Ca levels below 7 mg / dl when symptomatic and 6 mg / dl when no symptoms
or the amount of ionized Ca is less than 2.5-3 mg / dl.
Symptoms; hypoclasemia impairs muscle contraction and nerve conduction by lowering the threshold
potential. FAQ's
most important symptoms; convulsions, loss of consciousness and apnea. Hidden in non-severe
hypocalcemia
Chvostek and Trousseau (+) during the tetani period.
Hypocalcemia heart block, ventricular fibrillation, hypotension, heart failure, QT prolongation in ECG
it may cause apnea and hypoxia, especially in newborns. Laryngospasm may occur.
If hypocalcemia becomes chronic (rickets, hypoparathyroidism), demineralization of the bone occurs,
resulting in bone
pains and fractures may occur. Cataract occurs if hypoclasia lasts more than 1 year.
Other findings; the skin dries, becomes rough and peeled. Sparse hair, nail fracture and teeth
disorders are seen.
(Answer D)
6. Which of the following is seen in preterm infants who have undergone TPN for a long time
complications are not? (September 2004)
A) Cholestatic jaundice
B) Hyperlipidemia
C) Alkalose
D) Nephrocalcinosis
E) Hyperammonemia
Complications after preterm TPN administration;
one. Osmotic diuresis
2nd. azotemia
3. hyperammonemia
4. hypoglycemia
5. nephrocalcinosis
6. Cholestatic jaundice
7. hyperlipidemia
(Answer C)
7. Which of the following has not increased in diabetic mothers?
(September 2004)
A) Transposition of the great arteries
B) Lumbosacral agenesis syndrome
C) Holoprosencephaly
D) Duodenal atresia
E) Isolated pulmonary atresia
ALL TYPES OF TISTIME QUESTIONS
55
PEDIATRICS
See description of question 3
DIABETIC MOTHER BABY (DAB): Diabetes mothers type 1, type 2 and gestational diabetes
has negative effects on
Increased risks in diabetic mothers: polyhydramniosis, preterm labor, pyelonephritis, chronic hypertension
increased risk of fetal loss (especially> 32 GH, diabetic ketoacidosis and congenital anomalies
What happens in those).
The main factor that initiates pathological events in the baby of diabetic mother is glucose elevation.
hyperinsulinism
the result of this develops.

file:///C|/Users/User/Downloads/mcq.htm[04-Aug-19 10:20:40 AM]


Congenital anomalies are caused by periconceptional glucose elevation. Control is prevented.
Acidosis due to the relative deficiency of insulin in the early period leads to congenital anomalies.
3 times more than non-diabetic.
Findings due to fetal hyperglycemia:
• Congenital anomalies
• Decreased growth (0-20 GH, early pregnancy)
• Hyperinsulinemia (> 20 GH)
• Findings due to fetal hyperinsulinism:
• Hypoglycemia (0-7 days)
• Lack of surfactant
• Immature liver metabolism
Fetal hyperglycemia and hyperinsulinism-related findings
Fetal macrosomia (> 20 GH): labor asphyxia, transient tachypnea, cardiomyopathy
Fetal hypoxia (> 30 GH): Polycythemia, iron anomalies
The risk of stroke and renal vein thrombosis increases due to polycythemia.
Clinical manifestations are polycythemic infants, most of whom are born with LGA. All organs except the
brain are large
It happens. Canopy hairy (hypertrichosis pinna) can be seen
Hypoglycemia should be checked for blood sugar in the first hour.
Hypocalcemia and hypomagnesemia
Polycythemia and jaundice
Cardiomegaly occurs asymmetric septal hypertrophy and is confused with idiopathic subaortic stenosis.
RDS and tachypnea are more common.
(Answer E)
56
ALL TYPES OF TISTIME QUESTIONS
NEWBORN
8. In the presence of resistant hypocalcemia in the treatment of calcium in a newborn baby,
Which of the following should be considered? (September 2003)
A) Hypophosphatemia B) Hypomagnesemia
C) Hypopotasemia D) Hyponatremia
E) Hypouricemia
The classic finding of hypomagnesemia is hypocalcemic convulsion unresponsive to calcium treatment, such
as tetany.
the findings. Treatment is intramuscular administration of magnesium. (Intravenous administration of
calcium due to necrosis)
Generally, low magnesium is associated with hypocalcemia. Causes of hypomagnesemia:
Diabetic mother's baby
Blood exchange (due to binding of citrate, complete recovery takes 10 days) and total parenteral
insufficiency
Multiple pregnancies, preterm labor and intrauterine growth retardation Absorption disorders
Renal losses (amphotericin B, diuretics) Hypoparathyroidism, hyperphosphatemia
Hypermagnesemia: It occurs because the mother is given too much at birth. Reduction in absorption,
reflect-
Decreased labor, respiratory and nervous system depression, may cause meconium spigot syndrome.
Magnesium is abundant in antacids and laxatives. Their use in older children
hypermagnesemia. Treatment includes calcium administration and diuretics.
(Answer B)
9. Which of the following does not cause hypocalcemia in the neonatal period? (April 2000)
A) Maternal hypoparathyroidism B) Maternal diabetes
C) Perinatal asphyxia D) High phosphate cow milk intake
E) Hypomagnesemia
Causes of hypocalcemia in the newborn include:
one. Prematurity 2. Asphyxia, stress
3. Diabetic mother child 4. High phosphate load
5. Primary hypoparathyroidism 6. Maternal hyperparathyroidism
7. Pseudohypoparathyroidism 8. hypomagnesemia

file:///C|/Users/User/Downloads/mcq.htm[04-Aug-19 10:20:40 AM]


9. Renal diseases 10. Bicarbonate, citrated blood transfusion, furosemide, white light phototherapy,
intravenous
11th. Lipid administration
(Answer A)
ALL TYPES OF TISTIME QUESTIONS
57
PEDIATRICS
10. In the first 12 hours in a newborn with 38 weeks of 1600 gr, head circumference of 33 cm and 48 cm
which is the most common? (April 99)
A) Anemia B) Sclerodema
C) Respiratory distress syndrome D) Hyperbilirubinemia
E) Hypoglycemia
The infant described above is a term SGA infant. The most common reason for this is placental insufficiency
and this situation causes the baby to have hypoxia and
cause stress. Hypoxia causes erythropoietin increase and polycythemia. Sclerodem, on the other hand,
causes the subcutaneous fat tissue in infants with severe sepsis and impaired peripheral circulation.
It becomes a hard wood like esterification and heralds the bad progress. Respiratory distress syndrome is
caused by a lack of surfactant in preterm infants. At SGA
Increased stress due to cortisol stimulates the production of surfactant and protects against RDS.
Hyperbilirubinemia may occur due to an increase in erythrocyte mass, but it does not occur in the first 12
hours. (Within 24 hours
hypoglycemia is the most common finding due to the lack of storage. Problems and pathogenesis of SGA
babies Fetal death: Acidosis, hypoxia, infection, congenital anomalies. Perinatal asphyxia: Chronic fetal
hypoxia-acidosis, meconium aspiration, uteroplasental perfusion
reduction. Hypoglycemia: Low depot, reduced glyconeogenesis, increased need (hypoxia, hypothermia and
Relative large brain tissue) Dysmorphic features: Syndromes, genetic-chromosomal abnormalities,
oligohydramnios-related anomalies,
TORCH infection Polycythemia / hyperviscosity: Increased erythropoietin Decreased O2 consumption /
hypothermia: Hypoxia, hypoglycemia, fasting effect and lack of adipose tissue Lung bleeding and persistent
fetal circulation.
(Answer E)
11th. Which of the following is not a cause of hypoglycemia in the newborn? (April 98)
A) Diabetic mother
B) Severe erythroblastosis fetalis
C) Low birth weight
D) Salicylate poisoning
E) Phenylketonuria
Diabetic mother causes hypoglycemia due to hyperinsulinism. Pancreas of amino acids (mainly leucine)
released from erythrocytes that break down erythroblastosis fetalis
stimulates and causes hypoglycemia by causing pancreatic hypeplasia and hyperinsulinism. Hypoglycemia
occurs due to insufficient stores at low birth weight. Although salicylate poisoning is unlikely to occur in the
newborn, it may be hypog-
He can do high school. Phenyl ketonuria also does not show hypoglycemia.
(Answer E)
58
ALL TYPES OF TISTIME QUESTIONS
NEWBORN
12. Which diabetic mother does not have a child? (April 97)
A) Myocardial hypertrophy
B) Congenital anomaly
C) Subdural hematoma
E) Renal vein thrombosis
E) RDS
Subdural hematoma may be increased due to increased birth traumas.
(Answer C)
13. A child born weighing 5600 grams has hypoglycemia. Acidosis and ketonuria
is the diagnosis in a non-patient? (April 96)

file:///C|/Users/User/Downloads/mcq.htm[04-Aug-19 10:20:40 AM]


A) Hyperinsulinism
B) Glucagonoma
C) MSUD
D) Organic acidemia
E) Phenyl ketonuria
Macrosomia and hypoglycemia are associated with hyperinsulinism. Hypoglycemia, acidosis and ketonuria
occur in organic acidemias. Phenyl ketonuria is screened because there are no signs at birth. Early pyloric
stenosis
resembling vomiting. Hyperglycemia occurs because glucagonoma is the opposite hormone of insulin.
Hypoglycemia is not seen. MSUD: Branched chain alpha-ketoacid occurs due to lack of dehydrogenase
complex. Main organic acidemias MSUD: Valine, leucine, isoleucine MMA: Valine and isoleucine Propionic
acidemia: Valine and isoleucine Isovaleric acidemia: Leucine LGA (large for gestational age) in the question
is child. LGA babies Identify babies whose birth weight is greater than 90 p by the week of birth. Mortality
risk increases when birth weight exceeds 4000 g. Diabetic and obese mothers are common in infants. Risk
of birth traumas, congenital anomaly (especially heart) and mental retardation is increased. Causes of LGA:
Maternal diabetes (if no vascular complication occurs, SGA develops) Beckwith-Wideman syndrome Genetic
predisposition Recipient twin in twin to twin transfusion Hydrops fetalis Transposition of large vessels
Postmature
(Answer A)
ALL TYPES OF TISTIME QUESTIONS
59
PEDIATRICS
14. In a macrosomic baby, plethoric face, hypertrichosis pinna and hypoglycemia are detected.
Which of the following should be considered? (April 94)
A) Adrenal insufficiency
B) Desquamation of the skin
C) Congenital cytomegalovirus infection
D) Toxoplasma
E) Diabetic mother baby
See 3. Description of the problem
(Answer E)
60
ALL TYPES OF TISTIME QUESTIONS
NEWBORN
INFECTIONS
one. Which of the following congenital infections is not expected to develop microcephaly?
(September 2011)
A) Cytomegalovirus
B) Herpes simplex virus
C) Rubella
D) Toxoplasmosis
E) Mumps
This question seems to be one of the simplest questions asked in the relationship between microcephaly
and TORCH group infections. Because the only picture that is thought to be associated with mumps
infection is primary endocardial
It is fibroelastozis.
Start related anomalies Wide anterior fontanel
• Achondroplasia
• Osteogenesis imperfecta
• Cleidocanial dysostosis
• Picnodisostosis
• Russell-Silver syndrome
• 13-18-21 trisomies
• Apert syndrome
• Hallermann-Streiff send.
• Prematurity
• Intrauterine growth retardation

file:///C|/Users/User/Downloads/mcq.htm[04-Aug-19 10:20:40 AM]


• Hiptoridi
• Hydrocephalus
• Congenital rubella syndrome
• Vit. D-deficiency rickets
• Hypophosphatasia
Primary microcephaly
• Family (OR, OD)
• Down send.
• Edward send.
• Cri-du-chat send.
Cornelli de Lange send.
• Rubistein-Taybi send. ALL TYPES OF TISTIME QUESTIONS
61
PEDIATRICS
Smith-Lemli-Opitz send. Cases seen in small front fontanel:
1-Craniosynostosis,
• 2-Microcephaly
3- Congenital hyperthyroidism
4-Wormian bone anomaly Craniotabes: Physiological in the first 3 months.
• Osteogenesis imperfecta,
Rickets
• cretinism,
• cleidocranial disostosis
• lacunar skull
• Down syndrome Secondary microcephaly
• Radiation
• TORCH infection
• Fetal alcohol / hydantoin
• Meningitis / Encephalitis
• Malnutrition
• Metabolic (Hyperphenylalaninemia)
• Hyperthermia
• Hypoxic ischemic encephalopathy Transplacental infections in infants:
Growth retardation: CMV, Plasmodium, rubella, toxoplasmosis, Treponema pallidum, Trypanosoma cruzi,
VZV
• Congenital anatomical defects
• Neonatal organ involvement
• Late sequelae Congenital anatomical defects and causative organisms: Factors such as HIV, HBV, measles
do not cause anatomical defects.
• Congenital anatomical defect: - Cataract and heart defect: Rubella - Hydrocephalus: HSV, lymphocytic
choriomeningitis virus, rubella, toxoplasmosis - Calcification: CMV (periventricular), HIV, toxoplasmosis
(diffuse), T. cruzi - Extremity hypoplasia: VZV - Microsphere CMV, HSV, rubella, toxoplasmosis -
Microphthalmos: CMV, rubella, toxoplasmosis
• Late sequelae: - Convulsions: CMV, enterovirus, rubella, toxoplasmosis - Deafness: CMV, rubella,
toxoplasmosis - Tooth / skeleton: Rubella, T. pallidum - Endocrinopathy: Rubella, toxoplasmosis - Eye
pathologies: HSV, rubella, toxoplasmosis, T. cruzi, T. pallidum. VZV - Hepatitis: Hepatitis B - Mental
retardation: CMV, HIV, HSV, rubella, toxoplasmosis, T. cruzi, VZV - Nephrotic syndrome: Plasmodium, T.
pallidum
(Answer E)
62
ALL TYPES OF TISTIME QUESTIONS
NEWBORN
2nd. Related to congenital cytomegalovirus infections in newborns
which of the following statements is incorrect? (September 2011)
A) can cause progressive hearing loss
B) 90% of cases come with acute symptomatic findings at birth
C) May cause premature

file:///C|/Users/User/Downloads/mcq.htm[04-Aug-19 10:20:40 AM]


D) May cause purpuric rash
E) Chorioretinitis may cause
Congenital CMV infection is the most common TORCH infection. 95% of the cases are asymptomatic. In
symptomatic cases, choriorrheinitis, microcephaly, intracranial calcifications, hepato / splenomegaly, icter,
cholestasis, petechiae, anemia and hearing loss may be seen in the eye. Congenital CMV is involved in the
cause of prema- turity.
(Answer B)
3. Which of the following is the most common complication of mumps?
(April 2010)
A) Thyroiditis D) Pancreatitis
B) Nephritis E) Optical neuritis
C) Meningitis
The same question asked in April 2004. The most common complications of mumps are septic meningitis
(meningoencephalitis) and orchitis and epididymitis in postpubertal men. Of course, the answer would be
parotite if you were. Complications:
one. Nervous system complications: Meningitis is the most common complication that may or may not be
encephalitis. Most often occurs after 5 days of swelling of the diaper. It can be before, during and after.
Others: Transverse myelitis, factional paralysis, neurosensory deafness, aquaduct stenosis. 2nd. Pancreatitis
3. Orchitis, epididymitis. Atrophy can be seen but infertility is rare. It happens more often in adults.
oophoritis
rare. 4. Thyroiditis 5. Myocarditis: Makes it rare. It causes endocardial fibroelastosis in fetus. 6. Arthritis. It
holds more large joints and does not cause permanent damage. MUMPS: A RNA virus (Paramyxovirus) from
the paramyxoviridae group. There is only one serotype. It is transmitted by droplet. Most infectious is 2
days before swelling and 5 days after swelling. The incubation period is 12-25 days, usually 6-18 days.
Objectives: Salivary glands, nervous system, pancreas, testis are frequently involved. Less ovarian, thyroid,
synovial
tissue, heart, kidney and liver. Clinical findings: Parotid gland is bilateral, less unilateral swollen and painful.
Hyperemia may occur in the stenon canal mouth (second molar line). As the swelling progresses, the jaw
angle disappears and the ear is pushed up. The ear parotid axis is intact. Diagnosis is based on clinical
findings. Differential diagnosis: Parotitis should be made by other factors. HIV. Influenza, parainfluenza,
CMV, EBV and enteroviruses. Sjögren's syndrome, suppurative parotitis (echinacea S. aureus and pus
flowing through the canal) of collagenosis The treatment of mumps is symptomatic; There is no specific
treatment.
(Answer C)
ALL TYPES OF TISTIME QUESTIONS
63
PEDIATRICS
4. What is the most common complication of chickenpox? (April 2010)
A) Cerebellar ataxia
C) Pneumonia
B) Myocarditis
D) Nephrotic syndrome
E) Pancreatitis
The most common complication of chickenpox is secondary bacterial infections. Chickenpox in healthy
children
The second most common cause of hospitalization is neurologic complications. The most common
neurological complication is cerebellitis. Immunological response from pathogenesis and vasculitis
rum. Neurological findings may be manifested by eningoencephalitis, cerebellar ataxia, sudden onset
seizure, different levels of consciousness or neck stiffness.
Varicella-Zoster Virus. Herpes virus belongs to the family.
May cause primary, latent or recurrent infection. While chickenpox infection provides lifelong immunity, the
virus remains hidden throughout the life of nerve ganglion cells and reactivates to shingles. incubation
period is 10-21 days. Clinical symptoms
one. Chickenpox: It is a vesicular rash. Lesions begin as an itchy macule, papules, vesicles are formed.
Fluid in the vesicle becomes turbid, bellies and crusts heals in 1 week
The recovery in. All lesions are contagious until crusting. It is typical that there are lesions at different
stages at the same time and the lesions are more dense in the trunk.

file:///C|/Users/User/Downloads/mcq.htm[04-Aug-19 10:20:40 AM]


The lesions do not leave traces unless there is secondary bacterial infection. A slight hypopigmentation or
hyperpigmentation.
2nd. Infection seen in vaccines: 1/5 children spend. 3. Progressive varicella: It occurs in pregnant women,
newborns and adolescents and immune deficiencies.
It is rare in healthy children. There is widespread internal organ involvement and coagulopathy. Death is
most commonly caused by pneumonia within 3 days. It is also seen in asthmatic children taking inhaled
steroids.
4. Neonatal varicella: Risk of varicella in the mother 5 days before and 2 days after birth.
VZIG is given and acyclovir is started to the rash.
5. Congenital varicella: zigzag scars in the skin are characteristic skin findings. dermatomal
fits to distribution.
6. Herpes zoster (Shingles) It is rare in healthy children under 10 years of age. Intrauterine or <1 year old.
Children do not have symptoms such as numbness, burning before, after
Postherpetic neuralgia is rarely seen. Dermatomal involvement.
(Answer A)
64
ALL TYPES OF TISTIME QUESTIONS
NEWBORN
5. Which of the following is most common in congenital rubella syndrome (September 2008)
A) Patent ductus arteriosus
B) Transposition of large vessels
C) Ventricular septal defect
D) Atrioventricular block
E) Atrial septal defect
Transposition of the great arteries is common in diabetic mothers. Concomitant pulmonary stenosis or right
If there are anomalies such as aortic arch, it is also associated with Di George and CATCH syndrome.
CATCH (22 q deletion): Cardiac anomaly, abnormal face, thymic hypoplasia-aplasia, cleft (cleft palate),
hypoplasia. VSD is the most common congenital heart disease in children, while ASD is the most common
congenital heart disease in adults.
heart disease. Atrioventricular block in infants with lupus due to SS-A (anti Ro), SS –B (anti La) antibodies
It is seen. Patent ductus arteriosus is the most common cardiac pathology seen in congenital rubella.
We have prepared two tables with you, which is why we have a very frequently asked topic.
(Answer A)
Clinical manifestations of congenital rubella infection Rubivirus-related rubella is a relatively self-limiting
picture in childhood in adults
causes a heavier table.
What is important is that congenital rubella syndrome (CRS) is a severe disease involving almost all organs
and systems.
makes the table.
Clinical manifestations of congenital rubella: Neural type deafness is the most common finding.
The most common cardiac finding is PDA, followed by right pulmonary stenosis. In general, the most
common findings were eye findings (71%)> deafness> low birth weight> psychomotor retar-
dation> congenital heart (48%). The least sign of neonatal purpura (death is more and is due to
pneumonia).
The most common ocular finding is salt and pepper retinopathy. Its effect on vision is minimal. The most
serious eye finding
uni or bilateral cataract.
Neurological findings may be progressive. Mental motor retardation and behavioral problems, autism and
deafness are observed even in those who are not in the beginning. Rarely progressive rubella
panencephalitis
can also be.
ALL TYPES OF TISTIME QUESTIONS
65
PEDIATRICS
Late findings:
• Progressive panencephalitis
• Diabetes mellitus

file:///C|/Users/User/Downloads/mcq.htm[04-Aug-19 10:20:40 AM]


• Thyroid dysfunction
• Glaucoma and vision problems
• The information above is summarized from the Nelson 2007 table.
Pathological findings of congenital rubella
Nervous system: Chronic meningitis, parenchymal necrosis and calcification
Eye: Microphthalmia, cataract, iridocyclitis, ciliary body necrosis, glaucoma, retinopathy Ear: cochlear
hemorrhage
Cardiovascular: PDA, pulmonary stenosis, VSD, myocarditis Lung: endothelial necrosis and interstitial
pneumonia
Liver: Giant cell transformation, disruption of lobular structure, fibrosis, bile stasis, Kidney and adrenal
gland: Interstitial nephritis, cytomegaly in adrenal cortex
Bone: cartilage thinning, osteoid malformation and mineralization disorder
Skin: Erythropoiesis Thymus: Absence of germinal center and histiocytic reaction
6. Which is one of the clinical signs of intrauterine infections
It is not? (September 2007)
A) Low birth weight
B) Hepatosplenomegaly
C) Jaundice
D) Ambiguous genitalia
E) Intracranial calcifications
Transplacental infections lead to a wide variety of relatively overlapping findings. The answer to the
question
very clear but let's explain a little other options. Low birth weight: CMV, Rubella, toxo, Treponema pallidum
(syphilis), plasmodium and Trypana-
soma cruzi, Varicella zoster virus Hepatitis: CMV, enterovirus, HSV
Hepatosplenomegaly: CMV, Rubella, toxo, T. pallidum, Plasmodium, T. cruzi, HIV, enteroviruses
Hyperbilirubinemia is direct.
Intracranial calcification: CMV, Tokso, HIV, T. cruzi is due to. Ambigius genitalia is not seen due to
intrauterine infections.
For simpler and relatively specific findings for those who call it too complex, see subparagraph. Clinical
manifestations of these infections are indicated in a table and neonatal organ involvement,
late sequelae and congenital anatomic defects. The question, which makes the finding, can be in the form
of which is late sequelae.
Late sequelae in transplacental infection: convulsion, deafness, eye pathologies, mental retardation
Endocrinopathy Skeletal and dental anomalies
66
ALL TYPES OF TISTIME QUESTIONS
NEWBORN
Hepatitis: HBV-related. The above causes of hepatitis are organ involvement in the neonatal period.
Hepatitis of HBV is late sequelae. Attention..
Nephrotic syndrome
Cataract and heart defect: only rubella
Limb hypoplasia: VZV
Carditis: Rubella, T. cruzi, Coxackie
Osteitis, tooth and skeletal anomalies: Rubella and T. pallidum.
Rhinitis: T. pallidum (bloody runny nose), enteroviruses
Endocrinopathy: Rubella and toxoplasmosis
Nephrotic syndrome: Plasmodium and T. pallidum
(Answer D)
7. Microphthalmitis, myocarditis, sensorineural hearing loss, meningoencephalitis and patent duc-
microorganisms causing intrauterine infection with symptoms such as tus arteriosus.
What is the mechanism? (April 2007)
A) Toxoplasma gondii
B) Chlamydia trachomatis
C) Varicella zoster virus
D) Cytomegalovius
E) Rubella virus

file:///C|/Users/User/Downloads/mcq.htm[04-Aug-19 10:20:40 AM]


See 5. Part of the pathological findings of the question.
(Answer E)
8. Which is one of the most common causes of meningitis in newborn infants (0-1 months)
is not one of the bacteria?September 2006)
A) Group B streptococci
B) Group D streptococci
C) Staphylococcus aureus
D) Gram (-) enteric bacilli
E) Listeria monocytogenes
The question was a bit unfair.
Because we can clearly find the answer in Krugman.
Infection factors according to age
Neonatal period:
S. agalactia
E. coli (strain K1) and other gram negative enteric bacilli
L. monocytogenes
enterococci
ALL TYPES OF TISTIME QUESTIONS
67
PEDIATRICS
Infants and young children:
S. pneumonia
N. meningitidis
H. influenza (not vaccinated)
> Children over 5 years
S. pneumonia
N. meningitidis
Answer D (according to Krugman)
Nelson on page 798 The most common bacterial meningitis agents are GBS, E. coli and L. monocytogenes.
Others S. pneumonia and other streptococci, non-typable H. influenza, coagulase positive
and negative staphylococci (coagulase positive S. aureus), Enterobacter, Pseudomonas, T. palli-
dum and M. tuberculosis. Not directly called enterococci but other streptococci
(no problem if the author knows that the Enterococci are excluded from Group D).
In the neurology section where meningitis is described (p. 2513), for detailed information, refer to the same
page.
and common pathogens in groups B and D streptococci (enterococci), gram-negetive enteric bacilli
(E. coli, Klebsiella) and listeria and the most common ones are group B streptococci and E. coli.
It is written. None of the staphylococci. Enterococci.
After all this, meningitis can be a benefit to you.
Complication and prognosis in bacterial infections:
Complications of bacterial infection:
Endocarditis, septic emboli, septic arthritis, osteomyelitis and bone destruction. Recurrent bacteremia
is a rare complication.
Candidemia: Causes vasculitis, endocarditis, endophthalmitis and abscess.
Mortality is highest in gram negative and fungal sepsis.
Risk factors for death and disability: seizures lasting longer than 72 hours, coma, leukopenia, inotropic need

no.
Early complications of meningitis: Ventriculitis, abscess, cerebritis.
Late complications of meningitis: Hearing loss (the most common complication, half occurs), behavior
Disorder, motor disorder, cerebral palsy, seizures and hydrocephalus. In meningitis, early and late com-
CT is the imaging method used in plications.
Prevention of nosocomial infections: The most important factor is the hand with alcohol or antibacterial
soaps.
And washing.
(Answer C)
9. Which of the following is not a characteristic finding of fetal alcohol syndrome?

file:///C|/Users/User/Downloads/mcq.htm[04-Aug-19 10:20:40 AM]


(September 2005)
A) Organomegaly
B) Facial anomalies
C) Cardiac anomalies
D) Mental retardation
E) Growth retardation
68
ALL TYPES OF TISTIME QUESTIONS
NEWBORN
Characteristics of fetal alcohol syndrome (FAS):
Height, weight and growth retardation around head
Facial anomalies (short palpebral fissure, epicanthus, maxillary hypoplasia and micrognathia, flat filtration
and
thin upper lip)
Cardiac defect (septal defects)
Minor extremity and joint abnormalities (limitation of movement and abnormal hand lines)
Developmental retardation and mental retardation
FAS risk increase: It depends on maternal advanced age, low socioeconomic status and the amount of
drinking.
Pulmonary involvement, genitourinary involvement and gastrointestinal system involvement are primarily
absent. post-
One of the criteria in the natal period according to the dye showing that growth is chronically impaired
weighing is low.
(Answer A)
10. Which of the following is not considered in the differential diagnosis of neonatal sepsis? (April 2005)
A) Congenital adrenal hyperplasia
B) Hyperglycemia
C) Organic acidemia
D) Urea cycle defects
E) Necrotizing enterocolitis
Hyperglycemia is not a differential diagnosis of sepsis. However, hypoglycemia may produce sepsis-like
findings and
differential diagnosis.
Metabolic disorders with sepsis differential diagnosis:
hypoglycemia
Adrenal problems: Bleeding, adrenal insufficiency and congenital adrenal hyperplasia
Metabolic diseases: Urea cycle defects, organic acidemias, lactic acidosis and galactosemia
The new question can be replaced by another disease such as homocystinuria, phenylketonuria and
histidinemia.
Differential diagnosis causes of GIS: NEC, spontaneous bowel perforation or structural anomalies.
Many diseases are confused with newborn sepsis. Hypoplastic left most involved in cardiac diseases
It is the heart.
Severe diseases with differential diagnosis of neonatal sepsis
Cardiovascular congenital: Hypoplastic left heart, other structural abnormalities, pulmonary hypertension
Cardiovascular Acquired: Myocarditis, hypovolemic, cardiogenic shock, pulmonary hypertension (other
secondary diseases).
Hematologic: Purpura fulminans (caused by homozygous protein C deficiency), immune-mediated platelets
topeni / neutropenia, hereditary coagulation disorders, severe anemia, congenital leukemia
Respiratory system: RDS, aspiration pneumonia (amniotic fluid, meconium and gastric content), pulmonary
hypoplasia (diaphragmatic hernia), tracheoesophageal fistula and transient tachypnea
Neurological: hypoxic ischemic encephalopathy, intracranial hemorrhage, botulism, seizure
(Answer B)
ALL TYPES OF TISTIME QUESTIONS
69
PEDIATRICS
11th. Which of the following is a microorganism that does not cause congenital infection?
(September 2004)

file:///C|/Users/User/Downloads/mcq.htm[04-Aug-19 10:20:40 AM]


A) T. gondii
B) Rubella
C) Rubeola
D) CMV
E) T. pallidum
The TORCH group, in its classical form, includes T. gondii, Other's (syphilis), rubella, CMV and HSV. The
vaccine that can be prevented is rubella.
In addition, many other factors can cause intrauterine infection. Other viral infections such as HIV, HBV,
Parvo, varicella zoster have been introduced.
Tuberculosis from bacteria, plasmodium from parasites and T. cruzi agents can also be transmitted
intrauterine.
Rubella rubella (ella cık means reduction jewelry), rubeola is measles. Infants may be infected intrauterine,
intrapartum and postpartum.
Syphilis, toxoplasmosis, rubella, CMV, parvo virus B19, varicella are most commonly transmitted intrauterine
transplacental.
HIV, HBV, HSV are mostly transmitted during the passage through the birth canal. TB is most commonly
transmitted by postnatal contact. Intrauterine transition is the primary focus of the liver is serious
disease becomes the cause.
(Answer C)
12. The mother's use of moderate or large amounts of alcohol during pregnancy
Which does not lead to? (September 2003)
A) Growth retardation
B) Heart defects
C) Joint abnormalities
D) Genital system anomalies
E) Mental retardation
See 9. Description of the problem
(Answer D)
13. Which of the following is not one of the typical eye findings of rubella?
(April 2003)
A) Cataract B) Glaucoma
C) Retinopathy D) Strabismus
E) Microphthalmia
See 6. Question.
(Answer D)
70
ALL TYPES OF TISTIME QUESTIONS
NEWBORN
14. Which of the following is the most common cause of sepsis in the newborn? (April 2001)
A) H. influenza
B) S. pneumonia
C) L. monocytogenesis
D) Group B streptococcus
E) S. pyogenes
The most common causative agents in neonatal sepsis are Group B streptococci, S. agalactia and E. coli.
Empirical antibiotic therapy: Ampicillin + cefotaxime or gentamicin. Late sepsis
instead of aminoglycoside 3. Belt should be used.
Neonatal infections according to onset time
Early onset infections: Maternal origin. It spreads during or before birth.
The source is the maternal genital system. They go out in the first 7 days, mostly in the first 3 days. GBS in
first 24 hours
interest. Maternal obstetric complications and prematurity are common. Multisystem involvement occurs.
Early en-
pneumonia is common
Intrapartum antibiotic prophylaxis only prevents early GBS infections.
Late-onset infections: May be of the maternal genital system or environment. Seven - 30
interests in the day. Infections can be focal or multisystem. Late infections associated with meningitis

file:///C|/Users/User/Downloads/mcq.htm[04-Aug-19 10:20:40 AM]


is common.
Late late onset infections occur after 30 days. Prematurity or chronic problem
because of long-term hospitalization.
Factors in neonatal systemic infections:
Early onset maternal origin: GBS, E. coli, Listeria are the most common agents.
Late-onset maternal origin: GBS, L. monositogenes most common, only salmonella from gram negatives,
T. pallidum, M. tuberculosis are others.
Late onset community-acquired: S. pneumonia and E. coli are the most common agents.
Late onset hospital-acquired: Coagulase-negative staphylococci are the most common.
(Answer D)
15. Causes of increased risk of developing acute bacterial infection from newborn
which one is not? (September 2000)
A) Chorioamnionite in mother
B) Prematurity
C) Boy
D) Macrosomy
E) 5th minute apgar is below 6
ALL TYPES OF TISTIME QUESTIONS
71
PEDIATRICS
Maternal chorioamnionitis is a risk for early neonatal sepsis.
Prematurity and low birth weight are the greatest risk for sepsis.
It is 2 times higher in boys than in girls. However, the difference is not significant in low birth weight
infants.
Low APGAR is consistent with hypoxia. Increased need for intervention, caused by catheter, intubation
risk.
It should not be forgotten that the most important factor increasing the tendency to infections in the
newborn is prematurity and low
birth weight.
Possible reasons for this are;
Preterm labor caused by maternal infection
Intra-amniotic infection is inversely proportional with gestational age
Immature immune system
Invasive procedures have to be performed more often
Intrapartum antibiotic prophylaxis only prevents early GBS infections.
Risk factors for neonatal infections
Male gender
Low birth weight
Preterm birth
Complicated labor and multiple pregnancy
Chorioamnionitis and urinary tract infection, prolonged membrane rupture time
Immune deficiency
Galactosemia (E. coli)
Malformations (obstructive uropathy)
(Answer D)
16. It is seen healthy after birth but when suckling, vomiting, tachypnea at 3 days,
In the patient with lethargy and convulsions, which should be considered first
is not one of the situations you need? (September 1999)
A) Organic acidemia
B) Sepsis
C) Urea cycle enzyme defects
D) Homocystinuria
E) Meningitis
The first diagnosis should be considered in this patient is sepsis. Because it's more frequent than others.
Homocysteinuria is a congenital methionine disorder (deficiency of cystathionine synthase). Patients at birth

It is normal. Diagnosis is usually made after the age of 3 with lens subluxation (ectopia lentis).

file:///C|/Users/User/Downloads/mcq.htm[04-Aug-19 10:20:40 AM]


There are also skeletal disorders similar to Marfan. See 12. Description of the problem
(Answer D)
72
ALL TYPES OF TISTIME QUESTIONS
NEWBORN
17. What is the finding that is not in neonatal meningitis? (September 98)
A) Nape stiffness
B) Lethargy
C) Apnea
D) Cyanosis
E) Vomiting
Nonspecific findings during meningitis (fever, headache, mayalgia, weakness, fever, tachycardia and
hypotension)
signs of meningeal irritation.
Signs of meningeal irritation: Nuchal stiffness, Kernig and Brudzinski and back pain. fontanel
These findings may not be taken when open.
(Answer A)
18. What is empirical treatment for neonatal sepsis? (September 97)
A) Ceftazidime + aminoglycoside
B) Ampicillin + cefotaxime
C) Ampicillin + gentamicin
D) 3rd generation cephalosporin
E) Penicillin + vancomycin
Classical empirical therapy for ampicillin enterococci and listerias must be added (third)
Because they are naturally resistant to generations). Ampicillin + gentamicin if the other option is early
sepsis
can be used as. However, if late sespsis is accompanied by aminoglycoside due to accompanying meningitis

(not good for CSF). Third generation cephalosporin (cefotaxime is most appropriate) should be used.
Ceftazidime can be used if pseudomonas is considered.
(Answer B)
19. Which virus is not the cause of congenital infection? (April 97)
A) CMV
B) Rubella
C) HBV
D) Herpes virus
E) Route virus
Rota virus is the causative agent of diarrhea in children in winter. Acute gastroenteritis is the most common
causative agent in children.
(Answer E)
ALL TYPES OF TISTIME QUESTIONS
73
PEDIATRICS
20. Which of the following is the least common finding in neonatal meningitis? (September 95)
A) Lethargy
B) Nape stiffness
C) Absorption
D) Convulsion
E) General condition disorder
See explanation of question 17
(Answer B)
21. Feeding difficulty after three days in a normal born child
What would you think first if it evolved? (April 93)
A) Sepsis
B) Tracheoesophageal fistula
C) Imperforate anus
D) Hyaline membrane disease

file:///C|/Users/User/Downloads/mcq.htm[04-Aug-19 10:20:40 AM]


E) Congenital heart disease
Almost all of the newborn with feeding difficulty or reduced absorption in the form we are more familiar
with
disease as the first finding.
Respiratory distress syndrome is not expected in a healthy newborn. Hyaline membrane disease
(RDS) is a disease of preterms and respiratory distress begins immediately after birth. Trakeoözofa-
geal fistulas often coexist with atresia and the most common type is a proximal atresic distal fistula.
type. Polyhydramniosis occurs in antenatal history. Foamy secretion from mouth after birth
revenue. Only H-type fustula may develop in late stages and may cause respiratory complaints after
feeding.
increase is seen.
In the imperforated anus, meconium output is delayed.
(Answer A)
22. Which of the following does not occur in neonatal meningitis? (September 89)
A) Restlessness
B) Cyanosis
C) Apnea
D) Nausea and vomiting
E) Neck stiffness
Meningitis is often accompanied by late sepsis. Meningeal irritation may not be present when the fontanel is
open.
(Answer E)
74
ALL TYPES OF TISTIME QUESTIONS
NEWBORN
23. Hydrocephalus, diffuse intracranial calcification and chorioretinitis
Which of the following should be considered first in the newborn? (April 88)
A) Congenital syphilis
B) Congenital rubella infection
E) Hepatitis B
D) T. gondii
E) CMV
These three findings are classic findings of congenital toxoplasma infection. Similar findings in CMV
calcifications are typically periventricular.
(Answer D)
24. Which congenital disease is the most common cause of patent ductus arteriosus? (September 87)
A) Herpes simplex virus
B) Rubella (rubella)
C) Measles
D) Down syndrome
E) Toxoplasmosis
See 3. Question Initial findings according to systems in neonatal infections
General: Fever, heat irregularity (more important), decreased absorption, poor appearance, edema GIS:
Abdominal distention, vomiting, circle, hepatomegaly
Respiratory system: Apnea, dyspnea, tachypnea, withdrawal, cyanosis, flank breathing Renal: Oliguria
Cardiovascular system: pallor, variegated, cold skin, tachycardia, bradycardia, hypotension Central nervous
system: Irritability, lethargy, tremor, seizures, hypotonia, decreased reflexes, irregular breathing
num, fontanel tension and treble crying Hematologic: Jaundice (pronounced direct hyperbilirubinemia),
pallor, petechia, purpura, bleeding
(Answer B)
ALL TYPES OF TISTIME QUESTIONS
75
PEDIATRICS
SCREENING
one. Which of the following routine newborn screening programs in Turkey
is covered? (September 2007)
A) Alpha-1 antitrypsin deficiency

file:///C|/Users/User/Downloads/mcq.htm[04-Aug-19 10:20:40 AM]


B) Congenital hypothyroidism
C) Neuroblastoma
D) Tuberculosis
E) AIDS
In our country, routine neonatal screening is performed for congenital hypothyroidism, phenylketonuria,
while biotidi-
Naz deficiency was also included in the screening program. Scans are taken on the Guthrie card while the
baby is discharged. It is made with tandem mass from 1 drop of blood with extended neonatal screening
and fatty acid
Many diseases, including oxidation defects, aminoacidopathies, are screened. In screening for
hypothyroidism, only TSH cases are tried to detect high cases. TSH
pituitary or hypothalamus-induced hypothyroidism cannot be caught.
(Answer B)
2nd. The disease which is included in the routine screening of newborn in our country is as follows:
Which is? (April 91)
A) Tyrosinemia
B) Alkaptanuria
C) Phenylketonuria
D) Tay Sachs
E) Congenital adrenal hyperplasia
Phenyl ketonuria is the first screening metabolic disease. Congenital hypothyroidism and biotidinase
deficiency
routine screening in our country.
(Answer C)
76
ALL TYPES OF TISTIME QUESTIONS
GENETIC
GENETIC
one. Her intelligence was normal and she had long stature, aortic dilatation and lens dislocation.
Which of the following diseases should be considered in a child? (September 2011)
A) Marfan syndrome
B) Neurofibromatosis
C) Phenylketonuria
D) Glutaricaciduria
E) Hurler syndrome
Normal intelligence, long stature, lens dislocation and aortic dilatation are typical features of MARFAN
syndrome.
teristics. Homocystinuria is important in differential diagnosis.
Homocystinuria and Marfan Syndrome Differential Diagnosis Symptoms
Mental retardation rare findings, psychiatric disorders Cardiovascular system
77 Arterial and venous thrombi, flushing Dissecting aneurysm, mitral valve
prolapse, aortic dilatation
Marfan syndrome; OD passes, no mental retardation and epilepsy, aortic dissection rather than
thromboembolism
anomalies. The lens ectopia is upward.
(Answer A)
2nd. It was observed that the deviations in height were -2.5 in a 9-year-old girl who was brought with the
complaint of short stature. Physical examination of this child with disproportionate short stature dome
palate, low nape hairline,
mane neck and nipple separation is detected. What is the most likely diagnosis for this child?
A) Hypothyroidism
B) Silver-Russell syndrome
C) Turner syndrome
D) Klinefeltersendrom
E) Hypochondroplasia
ALL TYPES OF TISTIME QUESTIONS

file:///C|/Users/User/Downloads/mcq.htm[04-Aug-19 10:20:40 AM]

Das könnte Ihnen auch gefallen